[obm-l] Re: [obm-l] RE: [obm-l] Re: [obm-l] são dife rentes?

2009-10-07 Por tôpico Ojesed Mirror
Porque a probalibidade não é 1/2 independente de trocar ou não a porta ?

Qualquer que seja a primeira escolha, sempre ficarão duas portas, uma com o 
carro e outra com um bode para ser escolhida uma delas.
Trocando ou não, é sempre uma escolha entre duas portas fechadas, sendo uma 
vencedora e a outra não. o que daria 50% de chance.
Na realidade a escolha, será feita depois que uma das portas for aberta, trocar 
ou não significa escolher uma porta ou outra de duas.
Tem algum furo este argumento ou está faltando algum detalhe na definição ?

Ojesed.

  - Original Message - 
  From: Patricia Ruel 
  To: OBM 
  Sent: Monday, October 05, 2009 7:28 PM
  Subject: [obm-l] RE: [obm-l] Re: [obm-l] são diferentes?


  Olá Rogério!
  O fato de se questionar qual é a probabilidade, depois que uma porta já foi 
aberta não faz com que a probabilidade agora passe a ser de 50% (e não de 2/3 
como no problema abaixo)? Porque a porta aberta não entrou em questão, é como 
se ela nunca existisse (poderíamos até ter 50 portas abertas, isso não mudaria 
a probabilidade com uma troca de porta). Acho que depois que uma porta já está 
aberta, quando se pergunta se a medança de porta aumentaria a probabilidade é 
uma situação diferente da do problema abaixo e a resposta deveria ser: NÂO. 
Diferentemente de se ter, por exemplo, dois candidatos, um decidido a mudar 
(2/3 de ganhar) e outro decidido a não mudar (1/3 de ganhar).
  Alguém me disse que esse problema causou muita discussão nos EUA, durante 
muito tempo,  e pessoas respeitáveis divergiram de opiniões. Teria sido por 
causa desse detalhe na formulação do problema, transformando-o em dois 
problemas distintos?  
   Será que estou viajando?
  Desde já, meus agradecimentos pela atenção.
   
   Date: Sat, 3 Oct 2009 23:37:24 -0300
   Subject: [obm-l] Re: [obm-l] são diferentes?
   From: abrlw...@gmail.com
   To: obm-l@mat.puc-rio.br
   
   Ola' JSilva,
   eu nao vi diferenca sensivel entre os enunciados, mas vamos la'...
   No velho problema, quem muda de porta tem 2/3 de probabilidade de
   ganhar o carro, o que significa que, se não mudar de porta, tem apenas
   1/3 de chance.
   Portanto, se o candidato resolve mudar de porta, ele dobra sua chance
   de ganhar o carro.
   Ou seja, as situações continuam parecendo exatamente iguais.
   
   []'s
   Rogerio Ponce
   
   Em 03/10/09, JSilvajosimat...@yahoo.com.br escreveu:
Olá amigos da lista! Muito provavelmente este conhecido problema já deve 
ter
sido discutido nesta lista, mas estou frequentemente vendo uma sutil
variação dele e não acredito que a resposta seja a mesma. Gostaria de
ouvir a opinião de vocês sobre a seguinte discussão:
 No velho problema abaixo, quem está decidido a mudar de porta tem 2/3 de
probabilidade de ganhar o carro, pois para tanto é necessário e suficiente
que a sua primeira escolha seja uma porta onde há um bode.
   
Mas costumo ver a seguinte versão: uma das portas que contém um bode é
aberta e, depois disso, o apresentador pergunta se o candidato quer mudar 
de
porta. Se o candidato resolver mudar, ele dobra a probabilidade de ele
ganhar o prêmio?
   
Acredito que são situações distintas. O que vocês acham?
   
   
1) Em um programa de auditório, o convidado deve escolher uma dentre três
portas. Atrás de uma das portas há um carro e atrás de cada uma das outras
duas há um bode. O candidato ganhará o que estiver atrás da porta que
escolher. O procedimento para a escolha da porta é o seguinte: o convidado
escolheria inicialmente, em caráter provisório, uma das três portas. O
apresentador do programa, que sabe o que há atrás de cada porta, abre 
neste
momento uma das outras duas portas, sempre revelando um dos dois bodes. O
convidado agora tem a opção de ficar com a primeira porta que ele escolheu
ou trocar pela outra porta fechada.
Roberto e Rodrigo são dois candidatos que deverão participar do programa
esta tarde. Roberto está decidido a mudar de porta quando chegar a sua 
vez,
e Rodrigo está decidido a não mudar de porta. Um tem mais chances de 
ganhar
o carro do que o outro? Explique.
   
   
   
   


Veja quais são os assuntos do momento no Yahoo! +Buscados
http://br.maisbuscados.yahoo.com
   
   =
   Instruções para entrar na lista, sair da lista e usar a lista em
   http://www.mat.puc-rio.br/~obmlistas/obm-l.html
   =


--
  Você sabia que pode utilizar o Messenger de qualquer tipo de celular? Saiba 
mais. 

[obm-l] Traição numa ilha grega

2008-11-04 Por tôpico Ojesed Mirror
As mulheres de uma ilha grega sabem quais delas estão sendo traídas por seus 
perceiros, mas não sabem sobre si mesmas.
Se alguma delas tiver certeza da traíção de seu parceiro, tem o direito de 
cortar o mal pela raíz.
Elas não falam sobre este assunto entre si.
Um dia chega a Rainha nesta ilha e afirma que lá existe pelo menos um traidor e 
vai embora.
O que acontece depois disto ?

Ojesed.

Re: [obm-l] Concurso CMS-2008

2008-10-21 Por tôpico Ojesed Mirror

Obrigado a todos.
Vou pedir mudança no gabarito.

Ojesed.

- Original Message - 
From: Rogerio Ponce [EMAIL PROTECTED]

To: obm-l@mat.puc-rio.br
Sent: Monday, October 20, 2008 11:35 AM
Subject: Re: [obm-l] Concurso CMS-2008


Ola' pessoal,como foi mal escrita, a questao obriga que alguma coisa tenha 
que seradivinhada.Pudesse haver mistura, obviamente o menor numero de grupos 
seria 1(englobando todos os alunos), e a resposta seria 432 (conforme o 
Ralphja' apontou).Mas penso que eles pretendiam que os grupos, todos com o 
mesmo numerode participantes,  tivessem apenas meninos ou apenas 
meninas.Seguindo essa linha, para minimizar o numero de grupos e' 
necessariomaximizar o numero de participantes de cada grupo.Como o MDC entre 
264 e 168 e' 24, cada grupo tera' 24 alunos, numtotal de 11 grupos de 
meninos e 7 grupos de meninas.Assim , a resposta e' 24 = letra D.

[]'sRogerio Ponce.


2008/10/19 Ojesed Mirror [EMAIL PROTECTED]: Prova do Colégio Militar de 
Salvador para admissão ao 6o ano do ensino fundamental - 2008 Questão 06 
No colégio MATEMÁGICO existem 264 meninos e 168 meninas. Se grupos forem 
formados de maneira que todos eles fiquem com a mesma quantidade de meninos 
e a mesma quantidade de meninas, a quantidade de alunos (meninos e meninas) 
por grupo, de modo que se tenha o menor número de grupos, é: A-17, B-18, 
C-21, D-24, E-36. A resposta do gabarito, publicado hoje é letra B, mas 
meu filho achou que o correto seria a letra D. Vou entrar com um recurso 
para correção do gabarito, mas gostaria de saber a opinião dos professores 
desta lista, se existe alguma interpretação ou argumento que sustente a 
resposta oficial dada. Ojesed. 
= 
Instru�ões para entrar na lista, sair da lista e usar a lista em 
http://www.mat.puc-rio.br/~obmlistas/obm-l.htm!
l 
=

=
Instru��es para entrar na lista, sair da lista e usar a lista em
http://www.mat.puc-rio.br/~obmlistas/obm-l.html
= 


=
Instru��es para entrar na lista, sair da lista e usar a lista em
http://www.mat.puc-rio.br/~obmlistas/obm-l.html
=


Re: [obm-l] Concurso CMS-2008

2008-10-20 Por tôpico Ojesed Mirror
Caro Vidal, atente com mais cuidado para o enunciado.
O que se procura é minimizar o número de grupos e não maximizar.


Grato, Ojesed.
  - Original Message - 
  From: *Vidal 
  To: OBM 
  Sent: Monday, October 20, 2008 2:13 AM
  Subject: Re: [obm-l] Concurso CMS-2008


  Caro Ojesed ou Desejo,

  O gabarito está correto (letra B).

  Para que todos os grupos tenham o *mesmo* número de meninos e meninas, o 
número de grupos tem que ser um *divisor comum* de 264 e 168.

  E para que o número de grupos seja o *menor* possível, este *divisor comum* 
deve ser *máximo*.

  Daí o *máximo divisor comum*:
  m.d.c. (264,168) = 24

  Entretanto, 24 é o número de grupos, e não o número de alunos em cada grupo, 
que é o pedido da questão.

  Para obtê-lo, basta calcular quantos meninos e quantas meninas comporão cada 
grupo:

  meninos : 264 / 24 = 11
  meninas : 168 / 24 = 7

  Logo, haverá 11 meninos e 7 meninas, isto é, 18 alunos em cada um dos 24 
grupos.

  Abraços,
  Vidal.

  :: [EMAIL PROTECTED]




  2008/10/20 dnasimento [EMAIL PROTECTED]

Bem, esse é um problema clássico de mdc, fatorando o número 264, 
encontramos 2³.3.17 e fatorando o número 168, encontramos 2³.3.7, logo o mdc 
entre eles é 2³.3 = 24 letra D.
  De: [EMAIL PROTECTED] 

  Para: obm-l@mat.puc-rio.br 

  Cópia:  

  Data: Sun, 19 Oct 2008 20:21:58 -0300 

  Assunto: [obm-l] Concurso CMS-2008 

 Prova do Colégio Militar de Salvador para admissão ao 6o ano do ensino 
 fundamental - 2008
 
 Questão 06
 No colégio MATEMÁGICO existem 264 meninos e 168 meninas. Se grupos forem 
 formados de maneira que todos eles fiquem com a mesma quantidade de 
meninos 
 e a mesma quantidade de meninas, a quantidade de alunos (meninos e 
meninas) 
 por grupo, de modo que se tenha o menor número de grupos, é: A-17, B-18, 
 C-21, D-24, E-36.
 
 A resposta do gabarito, publicado hoje é letra B, mas meu filho achou que 
o 
 correto seria a letra D.
 Vou entrar com um recurso para correção do gabarito, mas gostaria de 
saber a 
 opinião dos professores desta lista, se existe alguma interpretação ou 
 argumento que sustente a resposta oficial dada.
 
 Ojesed. 
 
 =
 Instru??es para entrar na lista, sair da lista e usar a lista em
 http://www.mat.puc-rio.br/~obmlistas/obm-l.html
 =


Um abraço cordial,
Danilo do Nascimento da Silva



Re: [obm-l] Concurso CMS-2008

2008-10-20 Por tôpico Ojesed Mirror
Ralph, é isso mesmo, eles erraram o enunciado, por isto vou pedir a anulação da 
questão.
Só queria saber se tem alguma sutileza que não tinha percebido.

Ex. se considerarmos que não podemos misturar meninos e meninas num mesmo 
grupo, o que não está rigorosamente descartado no enunciado, pedir o menor 
número de grupos faz sentido e a resposta seria a letra D.

Estou procurando um argumento que sustente e resposta de meu filho, pois ao 
invés de pedir anulação seria correção de gabarito, e ele fecharia a prova.

Grato, Ojesed.
  - Original Message - 
  From: Ralph Teixeira 
  To: obm-l@mat.puc-rio.br 
  Sent: Monday, October 20, 2008 9:01 AM
  Subject: Re: [obm-l] Concurso CMS-2008


  Eh isso mesmo, o *MENOR* numero de grupos? Que tal 1 grupo soh, com todo 
mundo dentro? Se for isso, resposta 264+168=432, questao anulada...

  Agora, se for o *MAIOR* numero de grupos, ou o *MENOR* numero de alunos por 
turma, entao eu fico com a resposta do Vidal: mdc(264,168)=24 grupos, cada um 
com 11+7=18 alunos, letra B.

  Abraco,
   Ralph


  On Sun, Oct 19, 2008 at 9:21 PM, Ojesed Mirror [EMAIL PROTECTED] wrote:

Prova do Colégio Militar de Salvador para admissão ao 6o ano do ensino 
fundamental - 2008

Questão 06
No colégio MATEMÁGICO existem 264 meninos e 168 meninas. Se grupos forem 
formados de maneira que todos eles fiquem com a mesma quantidade de meninos e a 
mesma quantidade de meninas, a quantidade de alunos (meninos e meninas) por 
grupo, de modo que se tenha o menor número de grupos, é: A-17, B-18, C-21, 
D-24, E-36.

A resposta do gabarito, publicado hoje é letra B, mas meu filho achou que o 
correto seria a letra D.
Vou entrar com um recurso para correção do gabarito, mas gostaria de saber 
a opinião dos professores desta lista, se existe alguma interpretação ou 
argumento que sustente a resposta oficial dada.

Ojesed. 
=
Instru�ões para entrar na lista, sair da lista e usar a lista em
http://www.mat.puc-rio.br/~obmlistas/obm-l.html
=




[obm-l] Concurso CMS-2008

2008-10-19 Por tôpico Ojesed Mirror
Prova do Colégio Militar de Salvador para admissão ao 6o ano do ensino 
fundamental - 2008


Questão 06
No colégio MATEMÁGICO existem 264 meninos e 168 meninas. Se grupos forem 
formados de maneira que todos eles fiquem com a mesma quantidade de meninos 
e a mesma quantidade de meninas, a quantidade de alunos (meninos e meninas) 
por grupo, de modo que se tenha o menor número de grupos, é: A-17, B-18, 
C-21, D-24, E-36.


A resposta do gabarito, publicado hoje é letra B, mas meu filho achou que o 
correto seria a letra D.
Vou entrar com um recurso para correção do gabarito, mas gostaria de saber a 
opinião dos professores desta lista, se existe alguma interpretação ou 
argumento que sustente a resposta oficial dada.


Ojesed. 


=
Instru��es para entrar na lista, sair da lista e usar a lista em
http://www.mat.puc-rio.br/~obmlistas/obm-l.html
=


[obm-l] Re: [obm-l] Polinômios de variável complexa

2008-05-17 Por tôpico Ojesed Mirror
Ribamar, o método de Cardano/Tartaglia, resulta nas raizes de um polinômio de 
grau 3, sendo elas reais ou complexas.
  - Original Message - 
  From: J. R. Smolka 
  To: obm-l@mat.puc-rio.br 
  Sent: Thursday, May 15, 2008 10:06 AM
  Subject: Re: [obm-l] Polinômios de variável complexa


  Obrigado ao Ojesed pela idéia de fazer uma substituição de variável do tipo 
z=(x+1) para simplificar a análise. Deve ser útil. Mas não dá para aplicar 
Cardano diretamente, porque (repito) este é um polinômio de variável complexa. 
Cardano serve para resolver equações cúbicas de variável real (possivelmente 
válido até se os coeficientes forem complexos), que não é o caso aqui.

  Não é a primeira vez que esta confusão acontece. Será porque a variável usada 
é x (que induz a pensar em números reais) em vez de z (como é comum para 
números complexos)? Pensar em x como um vetor de coordenadas cartesianas 
(a,b) ou polares (|x|,arg(x)) ajuda o raciocínio.

  Para os que (ainda) se interessarem no problema, lembro que uma função de C 
em C tem como domínio todo o plano de Argand, e a imagem será pelo menos um 
subconjunto (não necessariamente contínuo) de todo o plano de Argand.

  Neste caso, como a função é um polinômio de grau 3, cada ponto x do plano 
domínio é mapeado para um ponto do plano imagem através das translações e 
rotações provocadas pela  potenciação de x e pela multiplicação de x por 
números reais.

  A questão inicial, então, é descobrir que região do plano de Argand pode 
possuir raízes de P(x)=0. Depois determinar a localização destes pontos nesta 
região (em função de k, que é um número real). E, finalmente, analisar a figura 
geométrica descrita pelo deslocamento destes pontos no plano de argand quando k 
varia entre 0 e +inf.

  Exemplo do raciocínio da primeira parte: não existe x tal que P(x)=0 na 
região do plano de Argand definida por 0=arg(x)pi/4 porque neste caso 
im(x)0, im(x^2)0 e im(x^3)0, o que torna impossível que im(P(z))=0.

  Como disse antes, consigo enxergar as regiões do plano de Argand definidas 
por arg(z)=pi/2 (o semi-eixo imaginário positivo, excluída a origem) e por 
arg(z)=pi (o semi-eixo real negativo, também excluída a origem) como candidatas 
a hospedeiras das raízes de P(x)=0. Mas será que a minha visão geométrica está 
correta e completa?

  Ainda não desenvolvi a álgebra destes casos para verificar se um, outro ou 
ambos são compatíveis com P(x) (afinal de contas, estou fazendo isto por puro 
diletantismo, e o tempo livre para raciocinar livremente anda meio curto ;-)). 
Mas continuo interessado em idéias a respeito.

  [ ]'s


Esta questão foi da prova de álgebra do IME 1976/1977. Vou transliterar um 
pouco o enunciado.

Seja P(x)=(x+1)(x+3)(x+5)+k(x+2)(x+4), com x complexo e k real positivo. 
Desenhar no plano complexo o lugar geométrico  das raízes de P(x)=0 para todos 
os valores possíveis de k.

Tentei o seguinte: se z=a+bi é raiz de P(x), então P(z)=0, o que implica 
que Re[P(z)]=0 e Im[P(z)]=0, então daria para obter expressões em função de a e 
b que descrevessem o lugar geométrico procurado. Só que as expressões parecem 
intratáveis.

Alguma outra idéia?
  J. R. Smolka 


[obm-l] Re: [obm-l] Polinômios de variável complexa

2008-05-13 Por tôpico Ojesed Mirror
Smolka, pra facilitar faça w=x+3 que fica w^3 +kw^2 - 4w - 4 = 0.
Use Cardano pra ver que todas as raizes são reais.

Ojesed
  - Original Message - 
  From: J. R. Smolka 
  To: obm-l@mat.puc-rio.br 
  Sent: Tuesday, May 13, 2008 9:56 AM
  Subject: Re: [obm-l] Polinômios de variável complexa


  Primeiramente obrigado à Alane e ao Ralph pelas sugestões. Vamos por partes:

  A Alane lembrou que se z é uma raiz do polinômio, então o conjugado complexo 
de z também será raiz. Não tenho certeza absoluta, mas acho que este princípio 
se mantém para funções polinomiais de C em C.

  O Ralph fez uma análise como se o polinômio fosse função de R em R, que não é 
o caso. Mas me deu algumas idéias sobre como atacar o problema. Até agora estou 
apenas no nível qualitativo. Depois vou tentar resolver a álgebra (a menos que 
alguém me mostre que esta linha de raciocínio não tem futuro :-)). O que estou 
pensando é:

  1) Se k=0, P(x) tem três raízes reais em x=-1, x=-3 e x=-5.

  2) Deve existir uma faixa de valores 0k=k1 para a qual P(x) ainda apresenta 
três raízes reais, que vão excursionar em algum trecho do semi-eixo real 
negativo. A investigar: (a) Qual o valor de k1? (estudo de 
máximos/mínimos/inflexões via P'(x)=0 deve ajudar nisso); (b) qual(is) 
intervalo(s) do semi-eixo real negativo é(são) percorrido(s) pelas raízes?

  3) Se kk1 então deve continuar a existir uma raiz real (que também 
excursiona no semi-eixo real negativo) e um par de raízes complexas 
conjugadas. Sobre a raiz real a pergunta é: qual o seu intervalo de excursão? 
Sobre as raízes complexas o raciocínio é mais longo...

  4) Temos que P(x)=x^3+(k+9)x^2+(6k+23)x+(8k+15). Se z=r.e^(i.a) é raiz de 
P(x), então r^3.e^(i.3a)+(k+9)r^2.e^(i.2a)+(6k+23)r.e^(i.a)+(8k+15)=0. Então 
temos quatro componentes, com argumentos complexos 0 (número real), a, 2a e 3a. 
De cara enxergo como candidatos a raiz os números complexos na forma 
z=r.e^(i.pi/2), onde o valor de r depende de k. Desta forma, o componente de 
argumento complexo 2a=2.pi/2=pi  pode anular o componente de argumento complexo 
0, e o componente de argumento complexo 3a=3.pi/2 pode anular o componente de 
argumento complexo a=pi/2. Se isto realmente for possível (tenho que verificar 
a álgebra), então z excursiona em um intervalo do semi-eixo imaginário 
positivo, com este intervalo limitado em (pelo menos) um valor que é função de 
k1, e o seu conjugado complexo vai ter um comportamento espelhado no 
semi-eixo imaginário negativo.

  Então minha primeira visão (qualitativa) para o lugar geométrico procurado é: 
um conunto de intervalos (possivelmente contínuos ou parcialmente sobrepostos) 
no semi-eixo real negativo, um intervalo (talvez finito) no semi-eixo 
imaginário positivo e o seu espelho no semi-eixo imaginário negativo.

  Críticas? Sugestões?

  [ ]'s


Esta questão foi da prova de álgebra do IME 1976/1977. Vou transliterar um 
pouco o enunciado.
Seja P(x)=(x+1)(x+3)(x+5)+k(x+2)(x+4), com x complexo e k real positivo. 
Desenhar no plano complexo o lugar geométrico  das raízes de P(x)=0 para todos 
os valores possíveis de k.
Tentei o seguinte: se z=a+bi é raiz de P(x), então P(z)=0, o que implica 
que Re[P(z)]=0 e Im[P(z)]=0, então daria para obter expressões em função de a e 
b que descrevessem o lugar geométrico procurado. Só que as expressões parecem 
intratáveis.


  J. R. Smolka 


Re: [obm-l] Computaca Grafica!!!

2008-03-27 Por tôpico Ojesed Mirror
O assunto é interessante, mas não é para ser discutido nesta lista, que é de 
matemática olímpica.

O moderador vai puxar as orelhas de vocês.

Troquem logo seus emails pessoais e caiam fora numa boa.

Ojesed.

- Original Message - 
From: Eike Santos [EMAIL PROTECTED]

To: obm-l@mat.puc-rio.br
Sent: Thursday, March 27, 2008 11:08 AM
Subject: Re: [obm-l] Computaca Grafica!!!


Estudou sobre o modelo de Monte Carlo? e Photon Mapping?

Ats,
Marcos Eike

2008/3/27 Tiago Machado [EMAIL PROTECTED]:

sim. estudei na disciplina de processamento gráfico.

2008/3/27 Eike Santos [EMAIL PROTECTED]:


 Mas, vc tem estudado ou estudou Rendering?

 Ats,
 Marcos Eike

 2008/3/27 Tiago Machado [EMAIL PROTECTED]:



  Atualmente estou trabalhando na area de computação gráfica e técnicas
  interativas.
 
  2008/3/26 Eike Santos [EMAIL PROTECTED]:
 
 
   Blz. Rapaz... Tem msn?
   Trabalha em que área? Atualmente estou estudando Rendering!
  
   Ats,
   Marcos Eike
  
   2008/3/26 Tiago Machado [EMAIL PROTECTED]:
  
  
  
Oi Eike,
   
Faço IC na area de computação gráfica.
   
   
   
On Wed, Mar 26, 2008 at 7:09 PM, Eike Santos [EMAIL PROTECTED]
  wrote:



 Alguem esta trabalhando ou gosta de computacao gráfica em geral
aqui
 na lista? Vamos trocas info? Dúvidas e coisas afim?

 Ats,
 --
 Marcos Eike Tinen dos Santos
 +559881176565

 OpenSuse- Comunidade Open do Suse
 www.opensuse.org
 OpenSolaris- Comunidade Open do Solaris
 www.opensolaris.org


 
=
 Instruções para entrar na lista, sair da lista e usar a lista em
 http://www.mat.puc-rio.br/~obmlistas/obm-l.html

 
=

   
   
  
  
  
   --
  
  
  
   Marcos Eike Tinen dos Santos
   +559881176565
  
   OpenSuse- Comunidade Open do Suse
   www.opensuse.org
   OpenSolaris- Comunidade Open do Solaris
   www.opensolaris.org
  
  
=
   Instruções para entrar na lista, sair da lista e usar a lista em
   http://www.mat.puc-rio.br/~obmlistas/obm-l.html
  
=
  
 
 



 --



 Marcos Eike Tinen dos Santos
 +559881176565

 OpenSuse- Comunidade Open do Suse
 www.opensuse.org
 OpenSolaris- Comunidade Open do Solaris
 www.opensolaris.org

 =
 Instruções para entrar na lista, sair da lista e usar a lista em
 http://www.mat.puc-rio.br/~obmlistas/obm-l.html
 =







--
Marcos Eike Tinen dos Santos
+559881176565

OpenSuse- Comunidade Open do Suse
www.opensuse.org
OpenSolaris- Comunidade Open do Solaris
www.opensolaris.org

=
Instruções para entrar na lista, sair da lista e usar a lista em
http://www.mat.puc-rio.br/~obmlistas/obm-l.html
= 


=
Instruções para entrar na lista, sair da lista e usar a lista em
http://www.mat.puc-rio.br/~obmlistas/obm-l.html
=


Re: [obm-l] SigmaDelta

2008-03-23 Por tôpico Ojesed Mirror
Olá lista, considerando a importância que tem para mim a resolução desta 
questão, estou reformulando a pergunta na esperança de que alguem se interesse 
por ela.

Mostrar que a média aritimética dos valores gerador pela sequencia: D(1), 
D(2)D(n) converge para V quando R  |V|

onde D(n) = R * I(n) / |I(n)|, 

e I(n) = I(n-1) + V - D(n-1),

com: D(0) = I(0) = 0

R e V reais com R  0

Ojesed.



  - Original Message - 
  From: Ojesed Mirror 
  To: obm-l@mat.puc-rio.br 
  Sent: Friday, March 14, 2008 2:43 PM
  Subject: [obm-l] SigmaDelta


  Olá lista, peço ajuda, não achei uma saída analítica.

  A(n) = A(n-1) + ( D(n) - A(n-1) )/n

  onde:

  D(n) = sinal( I(n) )*R 
  e
  I(n) = I(n-1) + V - D(n-1)

  Mostrar que A(n) converge para V quando |V|  R

  Considerar: 
  A(0) = D(0) = I(0) = 0.
  n inteiro, R constante real positiva, V constante real.
  sinal() retorna 1 para argumento positivo, -1 para argumento negativo e 0 
para argumento nulo.


  Obrigado, Ojesed.

[obm-l] SigmaDelta

2008-03-14 Por tôpico Ojesed Mirror
Olá lista, peço ajuda, não achei uma saída analítica.

A(n) = A(n-1) + ( D(n) - A(n-1) )/n

onde:

D(n) = sinal( I(n) )*R 
e
I(n) = I(n-1) + V - D(n-1)

Mostrar que A(n) converge para V quando |V|  R

Considerar: 
A(0) = D(0) = I(0) = 0.
n inteiro, R constante real positiva, V constante real.
sinal() retorna 1 para argumento positivo, -1 para argumento negativo e 0 para 
argumento nulo.


Obrigado, Ojesed.

[obm-l] cossenos sucessivos

2007-12-20 Por tôpico Ojesed Mirror
Numa calculadora operando em radianos, submete-se qualquer número a operações 
sucessivas de cosseno e observa-se que o número converge para um mesmo valor, 
independente do valor inicial adotado.
O valor pode ser obtido numericamente de cos(x)=x.

Mostrar que esta operação transforma qualquer real neste número.

Ojesed

Re: [obm-l] Filhos semelhantes...

2007-12-19 Por tôpico Ojesed Mirror
Ruy, está errado, o correto seria (1/2)^46, pois são 23 cromossomos do óvulo e 
23 do espermatozoide.
O 1/2 vem do fato da mitose resultar sempre duas células a partir de uma.

Ojesed.
  - Original Message - 
  From: Bruno França dos Reis 
  To: obm-l@mat.puc-rio.br 
  Sent: Wednesday, December 19, 2007 12:23 PM
  Subject: Re: [obm-l] Filhos semelhantes...


  Isso é uma questão de matemática?
  O que vc quer dizer com sacanear esses caras? A primeira imagem que me vem 
na cabeça é vc furar os pneus dos carros deles, o que não é nada matemático 
(tampouco correto).


  2007/12/18, Ruy Oliveira [EMAIL PROTECTED]:
Dois amigos biólogos calcularam a probabilidade de uma
familia ter dois filhos ( não gemêos ) , totalmente
semelhantes. Eles , óbviamente consideraram os
aspectos biológicos para a ocorrencia de tal evento.
chegaram a resposta (1/2)^45... 
Alguém me ajudaria a sacanear esses caras??
Abraço




  Abra sua conta no Yahoo! Mail, o único sem limite de espaço para 
armazenamento!
http://br.mail.yahoo.com/ 
=
Instruções para entrar na lista, sair da lista e usar a lista em
http://www.mat.puc-rio.br/~obmlistas/obm-l.html 
=




  -- 
  Bruno FRANÇA DOS REIS

  msn: [EMAIL PROTECTED] 
  skype: brunoreis666
  tel: +33 (0)6 28 43 42 16

  e^(pi*i)+1=0 

RES: [obm-l] ANGULOS RETOS

2007-10-18 Por tôpico Ojesed Mirror
Arkon, 

O ponteiro dos minutos anda a 2pi por hora, o ponteiro das horas anda a 2pi
por 12 horas

Um ponteiro em relação ao outro anda a (2pi – 2pi/12) por hora ou seja a
11pi/6 por hora ou 22pi em 12 horas.

O que dá 11 voltas completas em 12 horas, totalizando 22 ângulos retos.

O que resulta finalmente em 44 ângulos retos em 24 horas.

 

Ojeded

 

De: [EMAIL PROTECTED] [mailto:[EMAIL PROTECTED] Em nome
de Fernando A Candeias
Enviada em: quinta-feira, 18 de outubro de 2007 08:13
Para: obm-l@mat.puc-rio.br
Assunto: Re: [obm-l] ANGULOS RETOS

 

Oi Arkon,

O Nehab me alertou de que minha solução está errada. Achei então o número
44, que entretanto não figura nas opções. Usei a força bruta, girando os
ponteiros de um relógio e contando. ( na verdade contei até doze horas e
multiploiquei por dois). 

Agora, por exclusão, a solução b) pode ser eliminada por claramente
insuficiente. A solução tem que ser par, pois o número da ângulos em 24 h
tem que ser o dobro do número obtido em doze horas. Isso exclui e). Mas a
solução para 12 horas não pode ser impar, pois então a de seis horas seria
fracionária, por simetria. Isso leva a necessidade de soluções que sejam
múltiplos de 4, portanto excluindo d) e dexando no páreo apenas   48 ( 44 se
tivessem colocado a opção certa) ou 24. 

Para decidir entre as duas o caminho seria calcular o angulo percorrido por
cada ponteiro em 24 horas a partir da velocidade angular, que temos, e
dividir a diferença por pi/4 para decidir entre 44 ou 24.



 

Em 17/10/07, Fernando A Candeias HYPERLINK
mailto:[EMAIL PROTECTED][EMAIL PROTECTED] escreveu: 

Um angulo reto é obtido a partir de quando os ponteiros coincidem ou de
quando estão em oposição. Situações que acontecem  a cada volta completa do
ponteiro grande. Como ele deverá dar 24 voltas os ponteiros farão 48 angulos
retos. Parece então que a opção é  a). 

Em 17/10/07, arkon HYPERLINK mailto:[EMAIL PROTECTED] [EMAIL PROTECTED]
escreveu: 

Alguém pode, por favor, resolver esta:

 

(EMMRJ-71) Os ponteiros de um relógio (das horas e dos minutos) em 24 horas
formam:

 

a) 48 ângulos retos.

b) 4 ângulos retos.

c) 24 ângulos retos.

d) 22 ângulos retos.

e) 23 ângulos retos.

 

DESDE JÁ MUITO OBRIGADO






-- 

 

No virus found in this incoming message.
Checked by AVG Free Edition.
Version: 7.5.488 / Virus Database: 269.15.0/1077 - Release Date: 18/10/2007
09:54


No virus found in this outgoing message.
Checked by AVG Free Edition. 
Version: 7.5.488 / Virus Database: 269.15.0/1077 - Release Date: 18/10/2007
09:54
 


[obm-l] Re: [obm-l] solução de equação complexa

2007-10-10 Por tôpico Ojesed Mirror
Kleber, faça por comparação:

fazendo Z= a+bi, temos que Z*=a-bi, então

a + bi + 2*(a - bi) = 1 - i

que nos dá:  a = 1/3 e b = 1
Z = 1/3 + i

Sds, Ojesed
  - Original Message - 
  From: Kleber Bastos 
  To: obm-l@mat.puc-rio.br 
  Sent: Wednesday, October 10, 2007 10:40 AM
  Subject: [obm-l] solução de equação complexa


  Não esotu lembrado como faço para resolver uma equação complexa do tipo :

  Z + 2*(conjugado de z ) = 1- i 

  -- 
  Kleber B. Bastos 

[obm-l] Re: [obm-l] Erro, diferenciais e taxa de variaçã o!

2007-10-04 Por tôpico Ojesed Mirror
A derivada direcional num ponto, é o produto interno do gradiente no ponto com 
o vetor unitário da direção dada.
  - Original Message - 
  From: saulo nilson 
  To: obm-l@mat.puc-rio.br 
  Sent: Wednesday, October 03, 2007 5:54 PM
  Subject: Re: [obm-l] Erro, diferenciais e taxa de variação!


  2) Suponha que a temperatura T em um ponto P(x,y) de um terreno é dada por 
T(x,y)=x*e^y - y*e^x graus Celsius. Se uma pessoa caminha nesse terreno, em um 
caminho reto que faz um ângulo de 3pi/4 com o eixo x positivo, como está 
variando a temperatura quando a pessoa está no ponto P(0,0)? Determine a taxa 
de variação da temperatura nesse ponto. 
  dy/dx=-1/rq2
  dt/dx=e^y+x*e^ydy/dx-dy/dx*e^x-y*e^x=1+1/rq2
  dt/dy=dx/dy*e^y+x*e^y-e^x-y*e^xdx/dy=-rq2-1
  gradT=(1+1/rq2;-1-rq2)
  ModulogradT=rq(1+2/rq2+1/2+1+2rq2+2)=rq(9/2+3rq2) oC/m

   
  On 10/3/07, Anselmo Sousa [EMAIL PROTECTED] wrote: 

Bom dia amigos. Gostaria de ajuda nos seguintes exercícios.
 
1) O ângulo de elevação de um ponto sobre o chão para o topo de um edifício 
é medido como sendo pi/6, com um erro relativo máximo de 0,03%. Suponha que a 
distância do ponto ao edifício é medida como sendo 100m, com erro possível 
máximo de 2cm. Use diferenciais para aproximar o erro relativo máximo na altura 
calculada do edifício. 
 
 
 
 
2) Suponha que a temperatura T em um ponto P(x,y) de um terreno é dada por 
T(x,y)=x*e^y - y*e^x graus Celsius. Se uma pessoa caminha nesse terreno, em um 
caminho reto que faz um ângulo de 3pi/4 com o eixo x positivo, como está 
variando a temperatura quando a pessoa está no ponto P(0,0)? Determine a taxa 
de variação da temperatura nesse ponto. 
 
 
 
 
Desde já, agradeço pela atenção desmedida.
 
Anselmo ;-)



Veja mapas e encontre as melhores rotas para fugir do trânsito com o Live 
Search Maps! Experimente já! 



[obm-l] Re: [obm-l] TRAJETÓRIA

2007-09-19 Por tôpico Ojesed Mirror
2^6=128 ???
  - Original Message - 
  From: Samir Rodrigues 
  To: obm-l@mat.puc-rio.br 
  Sent: Wednesday, September 19, 2007 1:01 PM
  Subject: Re: [obm-l] TRAJETÓRIA


  Como os passos devem ser dados no sentido positivo, ele sempre tem 2 
possibilidades pra cada passo, entao N = 2 ^ 6 = 128


  Em 19/09/07, Marcelo Salhab Brogliato [EMAIL PROTECTED] escreveu:
Olá Arkon, 

acredito que este seja um caso classico de: Quantas solucoes existem em 
x+y=a.
Neste caso, x+y=6...
tente com essa nova questao..

abraços,
Salhab

On 9/19/07, arkon  [EMAIL PROTECTED] wrote:



 Olá pessoal, alguém pode, por favor, resolver esta:



 (UFPB-84) Uma pessoa encontra-se na origem de um sistema ortogonal de 
eixos 
 0x e 0y. Ela deve dar seis passos no plano dos eixos, sendo cada um na
 direção de 0x ou de 0y, sempre no sentido positivo. De quantos modos pode
 ser feita a trajetória do percurso?

 

 a) 36.  b) 54. c) 128.  d) 64.e) 72.



 DESDE JÁ MUITO OBRIGADO

=
Instruções para entrar na lista, sair da lista e usar a lista em 
http://www.mat.puc-rio.br/~nicolau/olimp/obm-l.html
=




  -- 
  Samir Rodrigues 


--


  No virus found in this incoming message.
  Checked by AVG Free Edition. 
  Version: 7.5.487 / Virus Database: 269.13.25/1018 - Release Date: 19/9/2007 
15:59


Re: [obm-l] UFPB-77

2007-09-06 Por tôpico Ojesed Mirror
é um circulo de raio 2r.
  - Original Message - 
  From: arkon 
  To: obm-l 
  Sent: Thursday, September 06, 2007 1:27 PM
  Subject: [obm-l] UFPB-77


  Pessoal alguém, por favor, pode responder esta

   

  (UFPB-77) A união de todos os círculos de raio R, num mesmo plano, passando 
por um ponto fixo é:

   

  a) um ponto.  b) duas retas.  c) uma reta.  d) um círculo.  e) o plano.

 DESDE JÁ MUITO OBRIGADO

--


  No virus found in this incoming message.
  Checked by AVG Free Edition. 
  Version: 7.5.485 / Virus Database: 269.13.7/992 - Release Date: 6/9/2007 08:36


[obm-l] Re: [obm-l] CONCESSIONÁRIA

2007-08-30 Por tôpico Ojesed Mirror

2^8=128 

- Original Message - 
From: Marcelo Salhab Brogliato [EMAIL PROTECTED]

To: obm-l@mat.puc-rio.br
Sent: Wednesday, August 29, 2007 10:26 PM
Subject: Re: [obm-l] CONCESSIONÁRIA


Olá,

4 cores externas, 4 cores internas, 2 opcoes de marchas, 2 opcoes pro
ar, 2 opcoes pra direcao hidraulica, 2 para vidros e travas
eletricas..

deste modo, temos: 4*4*2*2*2*2 = 2^8 = 128..

abracos,
Salhab


On 8/29/07, arkon [EMAIL PROTECTED] wrote:




Pessoal alguém pode resolver , por favor, esta:



(MPE/TO-UNB) Uma concessionária oferece aos clientes as seguintes opções
para a aquisição de um veículo: 4 cores externas, 4 cores internas, 4 ou 5
marchas, com ou sem ar condicionado, com ou sem direção hidráulica, com ou
sem vidros e travas elétricas. Desse modo, são, no máximo, 128 as opções
distintas para a escolha de um veículo?



DESDE JÁ MUITO OBRIGADO.


=
Instruções para entrar na lista, sair da lista e usar a lista em
http://www.mat.puc-rio.br/~nicolau/olimp/obm-l.html
=


--
No virus found in this incoming message.
Checked by AVG Free Edition.
Version: 7.5.484 / Virus Database: 269.12.10/977 - Release Date: 28/8/2007 
16:29



=
Instruções para entrar na lista, sair da lista e usar a lista em
http://www.mat.puc-rio.br/~nicolau/olimp/obm-l.html
=


Re: [obm-l] Desigualdade, meninos e meninas... quase-off

2007-08-21 Por tôpico Ojesed Mirror
a desigualdade é válida para todo a e b real não nulo desde que tenham o nesmo 
sinal, podendo portanto serem ambos negativos tambem.
os menimos não viram isto ?

Ojesed.
  - Original Message - 
  From: Artur Costa Steiner 
  To: obm-l@mat.puc-rio.br 
  Sent: Tuesday, August 21, 2007 11:18 AM
  Subject: RES: [obm-l] Desigualdade, meninos e meninas... quase-off


  Eh, mas se puderem ser negativos a desigualdade nao eh valida. Os meninos 
aqui, incluinodo este aqui, menino do inicio dos anos 60, viram isso
  Artur   
-Mensagem original-
De: [EMAIL PROTECTED] [mailto:[EMAIL PROTECTED] nome de Carlos Eddy Esaguy 
Nehab
Enviada em: terça-feira, 21 de agosto de 2007 08:04
Para: obm-l@mat.puc-rio.br
Assunto: Re: [obm-l] Desigualdade, meninos e meninas... quase-off


Não resisti:

Pois então menina :-), sua apostila está errada...

Abraços,
Nehab, um menino, há muito e muito tempo...

At 04:43 21/8/2007, you wrote:

  Olá meninos, na minha apostila só fala que a e b são reais não nulos.

  -- 
  Bjos, 
  Bruna 


--


  No virus found in this incoming message.
  Checked by AVG Free Edition. 
  Version: 7.5.484 / Virus Database: 269.12.1/963 - Release Date: 20/8/2007 
17:44


[obm-l] Re: [obm-l] BETONEIRA e a média harmônica....

2007-08-10 Por tôpico Ojesed Mirror
Olá Nehab, você é um educador como poucos pois consegue tratar assuntos desde o 
mais elementar, como este, até os que estão na fronteira do conhecimento com 
uma clareza invejável.

Muitas vezes eu vejo assuntos sendo tratados aqui, que é uma lista aberta, com 
uma linguagem despreocupada em atingir o máximo de pessoas possível.

Eu fico na dúvida se estou diante de pessoas que estão somente exibindo 
conhecimento ou se os assuntos estão sendo tratados da forma mais palatável 
possível e eu é que estou muito defasado da turma. Geralmente fico com a 
segunda opção.

O fato é que uma minoria de pessoas geniais participam ativamente das 
discussões e a maioria observa.
Se isto não fere o objetivo maior da existência desta lista, desculpem minha 
preocupação.

Sds, Ojesed
  - Original Message - 
  From: Carlos Eddy Esaguy Nehab 
  To: obm-l@mat.puc-rio.br 
  Sent: Friday, August 10, 2007 9:00 AM
  Subject: Re: [obm-l] BETONEIRA e a média harmônica


  Oi, Ojesed ,

  At 02:04 10/8/2007, you wrote:

Seria correto dizer que se as massas não fossem iguais a resposta seria a 
média harmônica ponderada, com as massas sendo os ponderadores ?

  Sim, vale...  Veja: 

  A média harmônica das densidades, ponderadas pelas massas é, por definição: o 
inverso da média aritmética ponderada (pesos m1 e m2) dos inversos das 
densidades d1 e d2.  Ou seja:

  É o inverso de  [ m1 x (1/d1) + m2 x (1/d2) ]  /  (m1 + m2)   
  que vale (m1+m2) / [ (m1/d1 + m2/d2) ] 

  Mas esta expressão é exatamente a densidade média, pois é a massa total 
(m1+m2) dividida pelo volume total  (m1/d1 + m2/d2).

  Abraços,
  Nehab


  - Original Message - 

  From: Carlos Eddy Esaguy Nehab 

  To: obm-l@mat.puc-rio.br 

  Sent: Thursday, August 09, 2007 2:17 AM

  Subject: Re: [obm-l] BETONEIRA


  Oi, Arkon, Ponce e Desejo...


  Já que o Desejo (Ojesed Mirror) deu esta ótima resposta, fica aqui uma 
dica, pois problemas desta natureza já apareceram diversas vezes por aqui...


  Quando se introduz o conceito de médias (mesmo na 6 ou 7 séries) é 
extremamente oportuno sugerir contextos onde elas ocorrem (para não parecer um 
negócio artificial) e a resposta do Ojesed mostra que ele já adquiriu a malicia 
que eu acho legal.  


  Na Física a média harmônica ocorre com freqüência, pois ela é usual em 
todas as situações onde a grandeza da qual se deseja calcular a média é o 
quociente entre duas variáveis  e vejamos:  Velocidade é distância / 
tempo...  Densidade é massa / volume, resistência = voltagem/ corrente ...   
Logo, se desejamos calcular velocidade média, densidade média, resistência 
equivalente, fatalmente a média harmônica entra na jogada (caso os valores das 
distâncias, volumes ou voltagens sejam iguais e isto ocorre na ligação em 
paralelo - como as voltagens não se somam, a resistência equivalente é o dobro 
da média harmônica...), posto que velocidade média = dist total / tempo total; 
densidade  final = massa total / volume total... e resistencia = mesma 
voltagem / corrente total 


  Vejamos um exemplinho  clássico (o outro é o do Arkon, posto que se 
misturam iguais quantidades de MASSA...)


  Você vai a 60 km por hora num trecho de estrada e no mesmo trecho volta 
a 90 km/h.  Qual sua velocidade média?


  Ora, você está querendo medir velocidade média, mas a variável chave, 
que é o tempo, está no denominador das velocidades e as duas distâncias, de ida 
e de volta são iguais ...


  Logo a velocidade média (vm) é a média harmônica. 


  Veja: vm =  distância total / tempo total = (d1 + d2) / (t1 + t2)   
(A)


  Ocorre que t1 =  d1 /v1  e  t2 = d2/v2


  Levando estas expressoes em (A)  voce obtem 

  dist total / tempo total = (x + x) /  [x/v1 + x/v2]  =  2v1.v2  (v1+v2)   
que é a média harmônica...  


  Abraços,

  Nehab



  At 22:39 8/8/2007, you wrote:

A densidade total é a média harmônica das densidades parciais. 
  - Original Message - 
  From: arkon 
  To: obm-l 
  Sent: Wednesday, August 08, 2007 10:49 AM 
  Subject: [obm-l] BETONEIRA


  Alguém pode resolver esta, por favor:


   
  Uma betoneira está sendo preparada para produzir concreto. Receberá 
iguais quantidades, em massa, de areia, cimento e brita de densidades iguais a 
1,5; 1,3 e 2,4, respectivamente. Calcular a densidade do concreto que vai ser 
produzido pela betoneira. Multiplique o resultado por 10 e despreze a parte 
fracionária, caso exista.


   
  DESDE JÁ MUITO OBRIGADO
--
   
  No virus found in this incoming message. 
  Checked by AVG Free Edition. 
  Version: 7.5.476 / Virus Database: 269.11.8/941 - Release Date: 
7/8/2007 16:06




No virus found

Re: [obm-l] BETONEIRA

2007-08-09 Por tôpico Ojesed Mirror
Seria correto dizer que se as massas não fossem iguais a resposta seria a 
média harmônica ponderada, com as massas sendo os ponderadores ?
  - Original Message - 
  From: Carlos Eddy Esaguy Nehab 
  To: obm-l@mat.puc-rio.br 
  Sent: Thursday, August 09, 2007 2:17 AM
  Subject: Re: [obm-l] BETONEIRA


  Oi, Arkon, Ponce e Desejo...

  Já que o Desejo (Ojesed Mirror) deu esta ótima resposta, fica aqui uma dica, 
pois problemas desta natureza já apareceram diversas vezes por aqui...

  Quando se introduz o conceito de médias (mesmo na 6 ou 7 séries) é 
extremamente oportuno sugerir contextos onde elas ocorrem (para não parecer um 
negócio artificial) e a resposta do Ojesed mostra que ele já adquiriu a malicia 
que eu acho legal.  

  Na Física a média harmônica ocorre com freqüência, pois ela é usual em todas 
as situações onde a grandeza da qual se deseja calcular a média é o quociente 
entre duas variáveis  e vejamos:  Velocidade é distância / tempo...  
Densidade é massa / volume, resistência = voltagem/ corrente ...   Logo, se 
desejamos calcular velocidade média, densidade média, resistência equivalente, 
fatalmente a média harmônica entra na jogada (caso os valores das distâncias, 
volumes ou voltagens sejam iguais e isto ocorre na ligação em paralelo - como 
as voltagens não se somam, a resistência equivalente é o dobro da média 
harmônica...), posto que velocidade média = dist total / tempo total; densidade 
 final = massa total / volume total... e resistencia = mesma voltagem / 
corrente total 

  Vejamos um exemplinho  clássico (o outro é o do Arkon, posto que se misturam 
iguais quantidades de MASSA...)

  Você vai a 60 km por hora num trecho de estrada e no mesmo trecho volta a 90 
km/h.  Qual sua velocidade média?

  Ora, você está querendo medir velocidade média, mas a variável chave, que é 
o tempo, está no denominador das velocidades e as duas distâncias, de ida e de 
volta são iguais ...

  Logo a velocidade média (vm) é a média harmônica. 

  Veja: vm =  distância total / tempo total = (d1 + d2) / (t1 + t2)   (A)

  Ocorre que t1 =  d1 /v1  e  t2 = d2/v2

  Levando estas expressoes em (A)  voce obtem 
  dist total / tempo total = (x + x) /  [x/v1 + x/v2]  =  2v1.v2  (v1+v2)   que 
é a média harmônica...  

  Abraços,
  Nehab


  At 22:39 8/8/2007, you wrote:

A densidade total é a média harmônica das densidades parciais.

  - Original Message - 

  From: arkon 

  To: obm-l 

  Sent: Wednesday, August 08, 2007 10:49 AM

  Subject: [obm-l] BETONEIRA


  Alguém pode resolver esta, por favor:





  Uma betoneira está sendo preparada para produzir concreto. Receberá 
iguais quantidades, em massa, de areia, cimento e brita de densidades iguais a 
1,5; 1,3 e 2,4, respectivamente. Calcular a densidade do concreto que vai ser 
produzido pela betoneira. Multiplique o resultado por 10 e despreze a parte 
fracionária, caso exista.





  DESDE JÁ MUITO OBRIGADO


--

  No virus found in this incoming message.

  Checked by AVG Free Edition. 

  Version: 7.5.476 / Virus Database: 269.11.8/941 - Release Date: 7/8/2007 
16:06



--


  No virus found in this incoming message.
  Checked by AVG Free Edition. 
  Version: 7.5.476 / Virus Database: 269.11.8/941 - Release Date: 7/8/2007 16:06


Re: [obm-l] BETONEIRA

2007-08-08 Por tôpico Ojesed Mirror
A densidade total é a média harmônica das densidades parciais.
  - Original Message - 
  From: arkon 
  To: obm-l 
  Sent: Wednesday, August 08, 2007 10:49 AM
  Subject: [obm-l] BETONEIRA


  Alguém pode resolver esta, por favor:

   

  Uma betoneira está sendo preparada para produzir concreto. Receberá iguais 
quantidades, em massa, de areia, cimento e brita de densidades iguais a 1,5; 
1,3 e 2,4, respectivamente. Calcular a densidade do concreto que vai ser 
produzido pela betoneira. Multiplique o resultado por 10 e despreze a parte 
fracionária, caso exista.



  DESDE JÁ MUITO OBRIGADO



--


  No virus found in this incoming message.
  Checked by AVG Free Edition. 
  Version: 7.5.476 / Virus Database: 269.11.8/941 - Release Date: 7/8/2007 16:06


[obm-l] Re: [obm-l] Ponto critico,função de duas variave is

2007-06-20 Por tôpico Ojesed Mirror
Faça o gradiente da função=0 que achará quatro pontos críticos.
Se não errei nas contas dá (0,1), (0,-1), (2i*raiz(3)/3, i*raiz(3)/6), 
(-2i*raiz(3)/3, -i*raiz(3)/6).

Ojesed 
  - Original Message - 
  From: dalyan castilho 
  To: tira duvidas ; olimpiada bm 
  Sent: Tuesday, June 19, 2007 9:34 PM
  Subject: [obm-l] Ponto critico,função de duas variaveis


  Oi pessoal!

  Não to conseguindo resolver esse exercicio,se alguem puder me ajudar.

  Ache os pontos criticos:

  f(x,y)= 4xy² - 2x²y - x

  Obrigada


--
  Novo Yahoo! Cadê? - Experimente uma nova busca. 

Re: RES: [obm-l] 2^x = x^2

2007-06-15 Por tôpico Ojesed Mirror
Um jeito é usando método numérico, a raiz é próxima de -0.74695962123 
usando o Matlab.
Interessante seria se alguém pudesse determinar analiticamente ou se provasse 
que assim não dá.

Ojesed.
  - Original Message - 
  From: Julio Sousa 
  To: obm-l@mat.puc-rio.br 
  Sent: Friday, June 15, 2007 11:13 PM
  Subject: Re: RES: [obm-l] 2^x = x^2


  eu já vi na HP que tem 3 raízes. Mas queria saber como chegar nelas de algum 
jeito. Abraço!


  On 6/15/07, Érica Gualberto Pongelupe [EMAIL PROTECTED] wrote: 
Oi Todo mundo

use um software de geometria dinâmica, por exemplo, o Cabri, ou mesmo um 
software do tipo Graphmatica que vc verá claramente as três raizes.

Abração

Érica

  Oi, Arthur (e Julio),

  Você esqueceu que x pode ser negativo.  Para x positivo, ok. Mas, 
faça um grafiquinho simples de y = x^2 e y = 2^x  e você veráque obviamente há 
uma raiz negativa (entre -1 e 0). 

  Abraços,
  Nehab 

  At 11:08 15/6/2007, you wrote:

Por inspecao, vemos que 2 e 4 sao raizesdesta equacao. Resta agora 
analisar se hah outras raizes. Temos 2^x = x^2se, e somente se, x ln(2) = 2 
ln(x), ou seja, sse ln(x)/x = ln(2)/2. Sejaa funcao definida em (0, oo) por 
f(x) = ln(x)/x. Temos que f'(x) = (1 -ln(x))/x^2, do que concluimos que f' se 
anula em x* = e.  A esquerdade e, f' eh positiva e, aa direita, eh negativa, o 
que nos mostra que fpassa por um maximo global em x* = e, para o quel f(x*) = 
1/e. Destaforma, f eh estritamente crecente m (0, e) e estritamente decrescente 
em(e, oo). Temos ainda que f eh continua, que lim x - 0+ f(x) = -oo eque lim x 
- oo f(x) = 0. Isso implica que, em (0, e) f assuma umaunica vez todos os 
reais em (-oo, 1/e) e que, em (e,oo) , assuma umaunica vez todos os reais em 
(1/e, 0).  Concluimos assim que , paraa0, a1/e, a equacao ln(x)/x = a tem 
exatamente duas raizes emR. Como ln(2)/2  1/e, ha exatamente 2 reais 
satisfazendo ln(x)/x= ln(2)/2. Logo, 2 e 4 sao as duas unicas raizes reais de 
2^x = x^2. 
 
Serah que hah outras raizes complexasnao reais?
 
Artur
 
 
 
 
 
 
 -Mensagem original-
De: [EMAIL PROTECTED] mailto:[EMAIL PROTECTED] nome de Julio Sousa
Enviada em: quinta-feira, 14 de junho de 2007 19:38
Para: obm-l@mat.puc-rio.br
Assunto: [obm-l] 2^x = x^2


  achar as raízes de 2^x = x^2



  -- 

  Atenciosamente

  Home Page: rumoaoita.com

  Júlio Sousa 
 
-- 



  -- 
  Atenciosamente
  Júlio Sousa 

Re: [obm-l] ITA-73

2007-02-12 Por tôpico Ojesed Mirror
Quando o triangulo estiver desdobrado teremos três bases de triângulos 
semelhantes:

1 - A base maior (do triângulo original, com área S1) que chamaremos de B1 e 
mede 12cm.
2 - A base do meio (onde foi dobrado) que chamaremos de B2 e é a medida que 
queremos achar.
3 - A base menor (do triângulo visível quando dobrado, com área S3) que 
chamaremos de B3.

Como os triângulos são semelhantes sabemos que: S3/S1 = (B3/B1)^2, de onde 
tiramos B3=12*sqrt(0.3).

Como B2 corta o trapézio formado por B1 e B3 ao meio, ela é a média aritmética 
entre B1 e B3.
Logo B2 = 6+6*sqrt(0.3).

Espero ter ajudado.
  - Original Message - 
  From: arkon 
  To: obm-l 
  Sent: Sunday, February 11, 2007 11:54 AM
  Subject: [obm-l] ITA-73


  Agradeço a todos que vem colaborando com resoluções de minhas questões 
enviadas anteriormente, muito obrigado mesmo.

   

  E se possível me enviem qual o macete para resolver a seguinte questão:



  (ITA-73) A base AB, de uma folha de papel triangular que está sobre uma mesa, 
mede 12 cm. O papel é dobrado levantando-se sua base, de modo que a dobra fique 
paralela à mesma. A área da parte do triângulo que fica visível após o papel 
ter sido dobrado vale 0,30 da área do triângulo ABC. O comprimento da dobra 
vale:

   

  a)  9,6 cm.   b) 9,4 cm.   c) 10 cm.   d) 8 cm.   e) N.D.A.



  Desde já agradeço muito.


Re: [obm-l] QUESTAO

2007-01-21 Por tôpico Ojesed Mirror
achei 10cm.
  - Original Message - 
  From: arkon 
  To: obm-l 
  Sent: Thursday, January 18, 2007 10:29 AM
  Subject: [obm-l] QUESTAO


  Olá, pessoal. Fiquei na dúvida no resultado desta questão. Alguém poderia me 
dizer o resultado, por favor?

  Um copo cilindro tem 6 cm de altura e tem uma circunferência da base medindo 
16 cm. Um inseto está do lado de fora do copo, a 1 cm do topo, enquanto, do 
lado de dentro, a 5 cm do topo, está uma gota de mel. A gota e o inseto 
encontram-se em geratrizes do cilindro que são simétricas em relação ao eixo do 
cilindro. A menor distância que o inseto deve andar para atingir a gota de mel 
é: 

  |

Re: [obm-l] log

2006-12-15 Por tôpico Ojesed Mirror
Se fosse isolar Y para obter X, poderia ser ?

xlogx = klogy - log(x^x) = log(y^k) - y^k = x^x - y = (x^x)^(1/k) = x^(x/k)  
com k=6.667

ojesed.


  - Original Message - 
  From: Cristian XV 
  To: obm-l@mat.puc-rio.br 
  Sent: Friday, December 15, 2006 10:01 AM
  Subject: [obm-l] log




   
  Tenho a seguinte equação e necessito isolar x para obter o valor de X como 
Fazer.
   
  X.LOG X = 6.667.LOG Y
  __
  Fale com seus amigos de graça com o novo Yahoo! Messenger 
  http://br.messenger.yahoo.com/ 


Re: [obm-l] Motivos para votar em LULA!!!

2006-10-22 Por tôpico Ojesed Mirror



não seria parcial ?

  - Original Message - 
  From: 
  Rafael Bonifácio 
  To: obm-l@mat.puc-rio.br 
  Sent: Sunday, October 22, 2006 3:24 
  AM
  Subject: RE: [obm-l] Motivos para votar 
  em LULA!!!
  
  Eu acho que isto é totalmente 
  OFF-TOPIC, e completamente imparcial.Meus pesames.Não entrei na lista 
  de matemática para discutir política, muito menos para ver coisas imparciais, 
  assim.
  
   Date: Sat, 21 Oct 2006 23:55:35 -0300 From: [EMAIL PROTECTED] 
  To: [EMAIL PROTECTED]; [EMAIL PROTECTED]; [EMAIL PROTECTED]; 
  [EMAIL PROTECTED]; [EMAIL PROTECTED]; 
  [EMAIL PROTECTED]; [EMAIL PROTECTED]; [EMAIL PROTECTED]; 
  [EMAIL PROTECTED]; [EMAIL PROTECTED]; [EMAIL PROTECTED]; 
  [EMAIL PROTECTED]; [EMAIL PROTECTED]; obm-l@mat.puc-rio.br 
  Subject: [obm-l] Motivos para votar em LULA!!!  Me limito a 
  citar vário links nos quais fundamento meu voto em LULA para  
  presidente. Leiam,  
  http://carosamigos.terra.com.br/da_revista/edicoes/ed114/valeapena.asp 
   
  http://forums.ecomm.com.br/cgi/dnewsweb.exe?cmd=articlegroup=forum.carosamigositem=7215utag= 
   
  http://forums.ecomm.com.br/cgi/dnewsweb.exe?cmd=articlegroup=forum.carosamigositem=7241utag= 
   
  http://forums.ecomm.com.br/cgi/dnewsweb.exe?cmd=articlegroup=forum.carosamigositem=7336utag= 
   
  http://forums.ecomm.com.br/cgi/dnewsweb.exe?cmd=articlegroup=forum.carosamigositem=7428utag= 
   
  http://www.adital.com.br/site/noticia.asp?lang=PTcod=24972 
   
  http://www.adital.com.br/site/noticia.asp?lang=PTcod=24993 
   
  http://www.adital.com.br/site/noticia.asp?lang=PTcod=24973 
   
  http://www.adital.com.br/site/noticia.asp?lang=PTcod=24969 
   
  http://www.adital.com.br/site/noticia.asp?lang=PTcod=24964 
   
  http://www.adital.com.br/site/noticia.asp?lang=PTcod=24963 
   
  http://www.adital.com.br/site/noticia.asp?lang=PTcod=24962 
   
  http://www.adital.com.br/site/noticia.asp?lang=PTcod=22858 
   O que acham?  
  ___  O Yahoo! está 
  de cara nova. Venha conferir!  http://br.yahoo.com  
  = 
  Instruções para entrar na lista, sair da lista e usar a lista em 
  http://www.mat.puc-rio.br/~nicolau/olimp/obm-l.html 
  =
  
  Expresse suas idéias de forma instantânea com o Windows Live Messenger! Windows Live Messenger! 


Re: [obm-l] Problema 4444..88..9

2006-10-12 Por tôpico Ojesed Mirror

O valor de n que dá como média 16 sem suprimir nenhum termo é 31.
Como a seqüência dada tem como média 16,1 significa que foi suprimido um 
termo na seqüência acima com valor inferior a 16.


Considerando k o termo suprimido tem-se:

( ( 1 + n ) n / 2 - k ) / ( n - 1 ) = 16,1

fazendo n=31 tem-se k=13

Ojesed.

- Original Message - 
From: Luís Lopes [EMAIL PROTECTED]

To: obm-l@mat.puc-rio.br
Sent: Wednesday, October 11, 2006 9:32 AM
Subject: Re: [obm-l] Problema ..88..9



Sauda,c~oes,

Oi Nehab,

Foi esta (PG) a solução que dei para o problema 84 do
meu livro sobre progressões. Concordo que a sua é
mais elegante.

Qual a solução do Lindski? E a do Sergio?

Segue o problema 46 do meu livro: suprimindo-se um
dos elementos do conjunto {1,2,.,n}, a média
aritmética dos elementos restantes é igual a 16,1.
Determine:

a) o valor de n;
b) o elemento suprimido.

[]'s
L.



From: Carlos Eddy Esaguy Nehab [EMAIL PROTECTED]
Reply-To: obm-l@mat.puc-rio.br
To: obm-l@mat.puc-rio.br
Subject: Re: [obm-l] Problema ..88..9
Date: Tue, 10 Oct 2006 15:05:07 -0300

Oi, Bruno,

Esqueci de dizer que a solução usual é por PG (ag).Chata, mas sai.
Se o número de algarismos de N é 2n, escreva o tal N assim (costumo errar 
contas, mas a idéia é a que segue):

N = 4*10^n*[10(n-1) +  + 1]  +  8*[ 10^(n-1) + ...+  1] + 9
N = 4*10^n[10^n -1] /9 + 8 *[10^n - 1]/9  + 9
Opere e obtenha
N =  4*10^(2n) + 2.4.10^n + 1 ] /9 =  (2*10^n + 1)^2 /9
Logo raiz(N) = (2*10^n + 1) /3

Abraços,
Nehab


At 13:33 10/10/2006, you wrote:

Oi Bruno,
O primeiro é do Lindski e já caiu no IME.   Acho minha solução 
bonitinha... :-)


Multiplique o tal cara N por 9 e perceba que 9N = 4...4.1 , ou 
seja,  9N = (200 + 1)^2...


Logo, o  N é quadrado e a resposta , 2001/3 = 667

Abraços,
Nehab

At 11:45 10/10/2006, you wrote:

Amigos peço ajuda para os seguintes problemas:

1)Demonstrar que os números 49 , 4489, 444889 obtidos colocando o número 
48 no meio do número anterior,são quadrados de números inteiros.


2) n retas paralelas de um plano se cruzam por uma série de m retas 
paralelas. Quantos paralelogramos podem ser separados na rede obtida ?


mais uma vez, obrigado.

Bruno


O Yahoo! está de cara nova. Venha 
http://us.rd.yahoo.com/mail/br/tagline/spirit/*http://br.yahoo.comconferir!



=
Instruções para entrar na lista, sair da lista e usar a lista em
http://www.mat.puc-rio.br/~nicolau/olimp/obm-l.html
=


--
No virus found in this incoming message.
Checked by AVG Free Edition.
Version: 7.1.408 / Virus Database: 268.13.3/473 - Release Date: 12/10/2006




=
Instruções para entrar na lista, sair da lista e usar a lista em
http://www.mat.puc-rio.br/~nicolau/olimp/obm-l.html
=


[obm-l] Re: [obm-l] dívidas do sete de setembro

2006-09-12 Por tôpico Ojesed Mirror



02) Sim.
A área lateral de um cone circular 
reto é dado por: SL=pi*r*g onde r=raio da base e g=medida da 
geratriz.
Para um cone equilátero temos 
r=g/2, o que resulta em SL=(pi*g^2)/2 que é a metade da área de uma 
circunferência de raio=g.

03) 8 raizes.


  - Original Message - 
  From: 
  ilhadepaqueta 
  To: obm-l 
  Sent: Thursday, September 07, 2006 4:37 
  PM
  Subject: [obm-l] dívidas do sete de 
  setembro
  
  
  Por gentileza, pode ser só o caminho das pedras.01) Seja a=1- i, 
  z complexo.a^2.z, a.z^2 e z^3 vértices distintos de um triangulo 
  equilatero.Pergunta-se quanto vale a parte imaginária de z.02) A 
  planificação de um cone equilátero dá a metade de uma 
  circunferência?03) Observando os extremos de y=6(senx)^2podemos 
  concluir q as raízes reais de 6(senx)^2=|x| são quantas?Obrigado mais 
  uma (3) vez
  
  

  No virus found in this incoming message.Checked by AVG Free 
  Edition.Version: 7.1.405 / Virus Database: 268.12.2/442 - Release Date: 
  8/9/2006


Re: Re: [obm-l] OBM Prova

2006-09-02 Por tôpico Ojesed Mirror

qualquer ajuda é válida.
se ele acredita que deus o abençoou diferentemente dos outros candidatos, 
certamente isto lhe dará uma maior serenidade na hora da prova, sendo 
verdade ou não.

claro que não elimina a necessidade de estudar muito.

- Original Message - 
From: [ Fabricio ] [EMAIL PROTECTED]

To: obm-l@mat.puc-rio.br
Sent: Saturday, September 02, 2006 7:13 PM
Subject: Re: Re: [obm-l] OBM Prova


Deus? Melhor estudar, não?



On 9/2/06, Alex pereira Bezerra [EMAIL PROTECTED] wrote:

boa sorte cara e que Deus te abençoe tb espero que vc consiga ir para
a segunda fase.Uma abraço

Em 01/09/06, Jefferson Franca[EMAIL PROTECTED] escreveu:
 Boa sorte para vc tbm.

 Saulo [EMAIL PROTECTED] escreveu:
 Saudações á todos do fórum!
 Queria desejar uma ótima prova amanhã para quem for fazer tanto do nível
 médio quanto o Universitário. Essa será a primeira vez que participo
 oficialmente dessa prova já que anteriormente tinha participado mas como
 convidado pela escola onde estudava que era o Elite, pois não era aluno
 do E.M. e então participei pq sempre tive vontade de participar e nunca
 tive oportunidade nas outras escolas.
 Hoje estou cursando Engenharia Mecânica e Matemática, 5° e 1°
 respectivamente e estou muito feliz por está participando, me preparei
 com algumas revista da Eureka em que meu professor Marcos do CEFET me
 deu, e livros de Cálculos e de Matemática que ganhei de amigos, e também
 por aqui.
 Gostaria de agradecer aos amigos que sempre enviavam questões, problemas
 e curiosidades sempre tentei buscar forças para resolver, ou então
 aguardar respostas dos amigos para ver se as minhas estavam pelo menos
 iguais. Realmente aqui na lista tem muitos feras, e por isso ficava meio
 que com receio de enviar questões e também enviar minhas soluções, mas
 isso já estou conseguindo vencer.
 Bem só não estou 100% feliz porque o único do CEFET q irá participar do
 Nível Superior será eu! Já que outros amigos desistiram de última hora
 de participar da prova alegando que essa prova não sirva para nada,
 confesso que fiquei entristecido com tais atitudes. Mas carrego minha
 cruz e sigo meu caminho,e espero realizar uma boa prova, como os demais
 amigos também.
 Um abraço, Boa Prova para todos, Deus nos abençoe.

 Saulo.


 
--


 =
 Instruções para entrar na lista, sair da lista e usar a lista em
 http://www.mat.puc-rio.br/~nicolau/olimp/obm-l.html
 =



 
 Yahoo! Search
 Música para ver e ouvir: You're Beautiful, do James Blunt



=
Instruções para entrar na lista, sair da lista e usar a lista em
http://www.mat.puc-rio.br/~nicolau/olimp/obm-l.html
=



=
Instruções para entrar na lista, sair da lista e usar a lista em
http://www.mat.puc-rio.br/~nicolau/olimp/obm-l.html
=


--
No virus found in this incoming message.
Checked by AVG Free Edition.
Version: 7.1.405 / Virus Database: 268.11.7/436 - Release Date: 1/9/2006

=
Instruções para entrar na lista, sair da lista e usar a lista em
http://www.mat.puc-rio.br/~nicolau/olimp/obm-l.html
=


[obm-l] Re: [obm-l] Série

2006-08-31 Por tôpico Ojesed Mirror
Use transformada Z para resolver a equação diferença, depois faça n ir ao 
infinito.


- Original Message - 
From: Josh Rodrigues [EMAIL PROTECTED]

To: obm-l@mat.puc-rio.br
Sent: Thursday, August 31, 2006 7:04 PM
Subject: [obm-l] Série



Olá, hoje encontrei o seguinte exercício numa apostila:

João pegou a calculadora de seu pai e começou a brincar, repetindo uma 
mesma seqüência de operações várias  vezes para ver o que acontecia. Uma 
dessas experiências consistia em escolher um número x1  qualquer, somar 5 
e dividir o resultado por 2, obtendo um novo número x2. A seguir ele 
somava 5 a x2 e dividia o resultado por 2, obtendo um novo número x3 . 
Repetindo esse processo, ele obteve uma seqüência de números


x1 , x2 , x3 , x4 , x5 ,., xn

Após repetir o processo muitas vezes, não importando com qual valor 
tivesse iniciado a seqüência de operações,
João reparou que o valor xn se aproximava sempre do mesmo número. Que 
número era esse?


É bem fácil ver que o número é 5 fazendo algumas contas. Mas eu gostaria 
de saber como que eu escrevo essa sequência e, de maneira mais rigorosa, 
mostrar que xn se aproxima sempre de 5.


Muito obrigado pela atenção.

_
O Windows Live Spaces é seu espaço na internet com fotos (500 por mês), 
blog e agora com rede social http://spaces.live.com/


=
Instruções para entrar na lista, sair da lista e usar a lista em
http://www.mat.puc-rio.br/~nicolau/olimp/obm-l.html
=


--
No virus found in this incoming message.
Checked by AVG Free Edition.
Version: 7.1.405 / Virus Database: 268.11.7/435 - Release Date: 31/8/2006




=
Instruções para entrar na lista, sair da lista e usar a lista em
http://www.mat.puc-rio.br/~nicolau/olimp/obm-l.html
=


[obm-l] Re: [obm-l] Re:[obm-l] Raizes quadrada e cúbica

2006-08-27 Por tôpico Ojesed Mirror

Usando o mesmo método para a raiz cúbica temos:
x_(n+1) = (x_n + a/x_n^2)/3

Genericamente temos:
x_(n+1) = (x_n + a/x_n^(i-1))/i, onde i é o índice da raiz e não precisa ser 
inteiro.


O melhor valor para iniciar a interação não sei.


- Original Message - 
From: claudio.buffara [EMAIL PROTECTED]

To: obm-l obm-l@mat.puc-rio.br
Sent: Sunday, August 27, 2006 10:59 PM
Subject: [obm-l] Re:[obm-l] Raizes quadrada e cúbica


Pra raiz(a), acho que poucos sao tao eficientes quanto:
x_(n+1) = (x_n + a/x_n)/2, comecando com, digamos x_1 = (1+a)/2.

Certamente eh mais eficiente do que aquele que parecia uma divisao e voce 
separava os algarismos do radicando em grupos de 2...


Naturalmente, por na mao, eu entendo sem computador mas com calculadora...

[]s,
Claudio.

-- Cabeçalho original ---

De: [EMAIL PROTECTED]
Para: obm-l@mat.puc-rio.br, obm-l@mat.puc-rio.br
Cópia:
Data: Sun, 27 Aug 2006 06:33:34 -0300
Assunto: [obm-l] Raizes quadrada e cúbica



   Alguém lembra os velhos algoritmos que nos permitiam calcular as
   raizes quadrada e cúbica na mão?


   Fabio




=
Instruções para entrar na lista, sair da lista e usar a lista em
http://www.mat.puc-rio.br/~nicolau/olimp/obm-l.html
=


--
No virus found in this incoming message.
Checked by AVG Free Edition.
Version: 7.1.405 / Virus Database: 268.11.6/428 - Release Date: 25/8/2006


=
Instruções para entrar na lista, sair da lista e usar a lista em
http://www.mat.puc-rio.br/~nicolau/olimp/obm-l.html
=


[obm-l] Re: [obm-l] Re: [obm-l] Re: [obm-l] Limite int eressantíssimo

2006-08-25 Por tôpico Ojesed Mirror

Errei novamente, é (4,0) mesmo.. valeu.

- Original Message - 
From: George Brindeiro [EMAIL PROTECTED]

To: obm-l@mat.puc-rio.br
Sent: Thursday, August 24, 2006 3:55 PM
Subject: RE: [obm-l] Re: [obm-l] Re: [obm-l] Limite interessantíssimo



Caro Ojesed,

Nos meus cálculos, R--4.
Creio que esteja correto, pois após encontrar a resposta verifiquei 
graficamente no winplot, pois realmente acreditava (devido à intuição, que 
nos deixa na mãos várias vezes), que R tendia ao eixo x por completo, como 
acredito foi sua primeira resposta.


Se quiser mandar a sua resolução, podemos constatar se houve algum erro, 
ou se o erro foi meu.


O problema não deixa de ser trivial, não há nada nele que não um pouco de 
trabalho manual.

Mas que o resultado é interessante.. isso é.

Abraços,
George B



From: Ojesed Mirror [EMAIL PROTECTED]
Reply-To: obm-l@mat.puc-rio.br
To: obm-l@mat.puc-rio.br
Subject: [obm-l] Re: [obm-l] Re: [obm-l] Limite interessantíssimo
Date: Thu, 24 Aug 2006 15:34:24 -0300

Errei nas contas, agora achei R-+2.
Informe se está certo pra eu mandar a demonstração.
Se tiver certo é realmente surpreendente !!! mas é trivial.



_
MSN Messenger: instale grátis e converse com seus amigos. 
http://messenger.msn.com.br


=
Instruções para entrar na lista, sair da lista e usar a lista em
http://www.mat.puc-rio.br/~nicolau/olimp/obm-l.html
=


--
No virus found in this incoming message.
Checked by AVG Free Edition.
Version: 7.1.405 / Virus Database: 268.11.5/426 - Release Date: 23/8/2006




=
Instruções para entrar na lista, sair da lista e usar a lista em
http://www.mat.puc-rio.br/~nicolau/olimp/obm-l.html
=


[obm-l] Re: [obm-l] Re: [obm-l] Limite interessantíssimo

2006-08-24 Por tôpico Ojesed Mirror



achei que convergia para 
(2,0)

  - Original Message - 
  From: 
  Rogerio Ponce 
  To: obm-l@mat.puc-rio.br 
  Sent: Thursday, August 24, 2006 4:42 
  PM
  Subject: RE: [obm-l] Re: [obm-l] Limite 
  interessantíssimo
  Tá errado, pois quando x=r^2/2 , entao y=sqrt(r^2 - 
  r^4/4)Usando a semelhanca de triangulos, obtemos a seguinte coordenada x 
  do ponto R:r * (r^2/2) / (r - sqrt(r^2 - r^4/4) ) , que converge 
  para 4.O ponto R converge para (4,0).[]sRogerio 
  PonceRogerio Ponce [EMAIL PROTECTED] 
  escreveu:
  Ola' 
George,Calculando o ponto Q: de C2 temos y^2 = r^2 - 
x^2Substituindo em C1, obtemos x=y=r^2/2Usando a semelhanca de 
triangulos para obtermos a coordenada x de R:r * (r^2/2) / (r - r^2/2) , 
que converge para 0 quando r-0Assim, o ponto R converge para a 
origem.[]sRogerio Ponce.George Brindeiro 
[EMAIL PROTECTED] escreveu:
É 
  fácil se deixar enganar pelas aparências meu caro..Não se engane! 
  Pense analiticamente.Abraços,George BFrom: 
  "Ojesed Mirror" Reply-To: 
  obm-l@mat.puc-rio.brTo: Subject: 
  [obm-l] Re: [obm-l] Limite interessantíssimoDate: Thu, 24 Aug 2006 
  01:55:57 -0300R- +oo- Original 
  Message - From: "George Brindeiro" To: Sent: 
  Wednesday, August 23, 2006 1:15 PMSubject: [obm-l] Limite 
  interessantíssimoCaros colegas de 
  lista,Não participo muito mandando problemas, 
  apenas observo suas soluções na maior parte do 
  tempo.Porém, me mandaram um problema em minha comunidade do 
  orkut, 'Cálculo Diferencial e Integral', que é muito 
  interessante, e nada trivial. Fiquei surpreso com o resultado! 
  Deleitem-se."Imaginem uma circunferência C1 com 
  equação (x- 1)²+y²=1 e outra circunferência C2, a ser 
  encolhida, com raio r e centro na origem. P é o ponto (0,r) , 
  Q é o ponto de intersecção superior das circunferências e R é 
  o ponto de intersecção da reta PQ com o eixo x.O 
  que acontecerá com R quando C2 encolher, isto é, quando 
  r---0+?"Minha solução está postada na 
  comunidade, se quiserem eu posso postá-la aqui 
  depois.Um Abraço,George 
  B._MSN 
  Messenger: instale grátis e converse com seus amigos. 
  http://messenger.msn.com.br=Instruções 
  para entrar na lista, sair da lista e usar a lista 
  emhttp://www.mat.puc-rio.br/~nicolau/olimp/obm-l.html=--No 
  virus found in this incoming message.Checked by AVG Free 
  Edition.Version: 7.1.405 / Virus Database: 268.11.5/426 - 
  Release Date: 
  23/8/2006=Instruções 
  para entrar na lista, sair da lista e usar a lista 
  emhttp://www.mat.puc-rio.br/~nicolau/olimp/obm-l.html=_MSN 
  Messenger: instale grátis e converse com seus amigos. 
  http://messenger.msn.com.br=Instruções 
  para entrar na lista, sair da lista e usar a lista 
  emhttp://www.mat.puc-rio.br/~nicolau/olimp/obm-l.html=
__Fale com seus 
amigos de graça com o novo Yahoo! Messenger 
http://br.messenger.yahoo.com/ 
  
  
  Yahoo! SearchMúsica para ver e ouvir: You're 
  Beautiful, do James Blunt
  
  

  No virus found in this incoming message.Checked by AVG Free 
  Edition.Version: 7.1.405 / Virus Database: 268.11.5/426 - Release Date: 
  23/8/2006


[obm-l] Re: [obm-l] Limite interessantíssimo

2006-08-23 Por tôpico Ojesed Mirror

R- +oo

- Original Message - 
From: George Brindeiro [EMAIL PROTECTED]

To: obm-l@mat.puc-rio.br
Sent: Wednesday, August 23, 2006 1:15 PM
Subject: [obm-l] Limite interessantíssimo



Caros colegas de lista,

Não participo muito mandando problemas, apenas observo suas soluções na 
maior parte do tempo.
Porém, me mandaram um problema em minha comunidade do orkut, 'Cálculo 
Diferencial e Integral', que é muito interessante, e nada trivial. Fiquei 
surpreso com o resultado! Deleitem-se.


Imaginem uma circunferência C1 com equação (x- 1)²+y²=1 e outra 
circunferência C2, a ser encolhida, com raio r e centro na origem. P é o 
ponto (0,r) , Q é o ponto de intersecção superior das circunferências e R 
é o ponto de intersecção da reta PQ com o eixo x.


O que acontecerá com R quando C2 encolher, isto é, quando r---0+?

Minha solução está postada na comunidade, se quiserem eu posso postá-la 
aqui depois.


Um Abraço,
George B.

_
MSN Messenger: instale grátis e converse com seus amigos. 
http://messenger.msn.com.br


=
Instruções para entrar na lista, sair da lista e usar a lista em
http://www.mat.puc-rio.br/~nicolau/olimp/obm-l.html
=


--
No virus found in this incoming message.
Checked by AVG Free Edition.
Version: 7.1.405 / Virus Database: 268.11.5/426 - Release Date: 23/8/2006




=
Instruções para entrar na lista, sair da lista e usar a lista em
http://www.mat.puc-rio.br/~nicolau/olimp/obm-l.html
=


Re: [obm-l] exercicio do ITA

2006-08-21 Por tôpico Ojesed Mirror



Considerando os limites da sombra 
do sol como um circulo com centro na origem do plano cartesiano, temos 
que:
X^2 + Y^2 = R^2 que é a equação de 
um circulo de raio R.
Daí temos que X = +-sqrt(R^2 
-Y^2)
Pelos dados do problema R=50 e Y = 
50-10=40, nos dando x=+-30
Então a faixa de sombra tem 
60km.
Se para uma faixa de 100km a 
duração é 3min, então para uma faixa de 60km a duração é 1min 48s.

Ojesed.

  - Original Message - 
  From: 
  carlos felipe ladeira 
  
  To: obm-l@mat.puc-rio.br 
  Sent: Monday, August 21, 2006 3:15 
  PM
  Subject: Re: [obm-l] exercicio do 
  ITA
  Não consegui entender como se chegou àqueles números... Mas a 
  resposta está correta, se puder fazer de novo agradeço.Ojesed 
  Mirror [EMAIL PROTECTED] escreveu:
  



x=+-sqrt(50^2 - 
40^2)
x=+-30
largura da 
faixa=60km
duração=1min 48s

  - 
  Original Message - 
  From: 
  carlos felipe ladeira 
  
  To: 
  obm-l@mat.puc-rio.br 
  Sent: 
  Sunday, August 20, 2006 9:53 PM
  Subject: 
  [obm-l] exercicio do ITA
  Alguem ai sabe resolver este aqui?( ITA - 73 ) 
  Durante o eclipse total do sol de março de 1970 a largura da faixa de 
  escuridão total foi de 100km. Em cada ponto do eixo central desta faixa, a 
  duração do período de escuridão total foi de 3 minutos. Qual foi a duração 
  deste período num ponto situado a 10km do limite da faixa de escuridão 
  total?(A) 1min36s(B) 
  1min48s(C) 1min30s(D) 
  0min36s(E) N.D.A.
  
  
  Yahoo! SearchMúsica para ver e ouvir: You're 
  Beautiful, do James Blunt 
  
  

  No virus found in this incoming message.Checked by AVG Free 
  Edition.Version: 7.1.405 / Virus Database: 268.11.3/423 - Release 
  Date: 18/8/2006
  
  
  Novidade no Yahoo! Mail: receba alertas de novas mensagens no seu celular. Registre 
  seu aparelho agora!
  
  

  No virus found in this incoming message.Checked by AVG Free 
  Edition.Version: 7.1.405 / Virus Database: 268.11.4/424 - Release Date: 
  21/8/2006


[obm-l] Re: [obm-l] dúvida

2006-08-20 Por tôpico Ojesed Mirror



Tem um erro na sua substituição da 
raiz, coloque 81 no lugar do segundo 27.

  - Original Message - 
  From: 
  cleber 
  vieira 
  To: obm-l@mat.puc-rio.br 
  Sent: Sunday, August 20, 2006 7:49 
  PM
  Subject: [obm-l] dúvida
  
  Olá amigos gostaria de saber da opinião de vocês sobre a seguinte 
  questão.
  
  Seja a1eea base dos logaritmos 
  neperianos, o valor de m para o qual a equação x^3 - 9x^2 + ( lna^m + 8)x - 
  lna^m = 0 tenha raízes em progressão aritmética,é dado por
  
  a) m = lna - 8 b) lna - 
  9c) m = 15/lna d) m = - 
  (9/8)*lna
  
  Resolvi da seguinte forma :
  Se as raízes estão em PA então ...
  b, b+r, b+2r são raízes.
  Usando Girard temos...
  b+b+r+b+2r = 9
  3b+3r = 9
  b+r = 3 que é uma raiz. Logo...
  
  27-27+ 3lna^m + 24 - lna^m = 0
  lna^m = -12 então m = -12/(lna) . Mas no gabarito consta letra 
  C
  
  
  
  Você quer respostas para suas perguntas? Ou você sabe muito e quer 
  compartilhar seu conhecimento? Experimente o Yahoo! 
  Respostas!
  
  

  No virus found in this incoming message.Checked by AVG Free 
  Edition.Version: 7.1.405 / Virus Database: 268.11.3/423 - Release Date: 
  18/8/2006


Re: [obm-l] exercicio do ITA

2006-08-20 Por tôpico Ojesed Mirror



x=+-sqrt(50^2 - 40^2)
x=+-30
largura da faixa=60km
duração=1min 48s

  - Original Message - 
  From: 
  carlos felipe ladeira 
  
  To: obm-l@mat.puc-rio.br 
  Sent: Sunday, August 20, 2006 9:53 
  PM
  Subject: [obm-l] exercicio do ITA
  Alguem ai sabe resolver este aqui?( ITA - 73 ) Durante 
  o eclipse total do sol de março de 1970 a largura da faixa de escuridão total 
  foi de 100km. Em cada ponto do eixo central desta faixa, a duração do período 
  de escuridão total foi de 3 minutos. Qual foi a duração deste período num 
  ponto situado a 10km do limite da faixa de escuridão 
  total?(A) 1min36s(B) 
  1min48s(C) 1min30s(D) 
  0min36s(E) N.D.A.
  
  
  Yahoo! SearchMúsica para ver e ouvir: You're 
  Beautiful, do James Blunt
  
  

  No virus found in this incoming message.Checked by AVG Free 
  Edition.Version: 7.1.405 / Virus Database: 268.11.3/423 - Release Date: 
  18/8/2006


Re: [obm-l] Invariantes

2006-08-15 Por tôpico Ojesed Mirror
Sei que este não é o tema central da discussão, mas se eu entendi bem a 
figura, dá pra desenhar-la sem tirar o lápis do papel.
Basta desenhar uma lateral da casa (um traço) e logo após desenhar a parte 
superior do telhado (dois traços), a partir daí qualquer caminho serve.

Esta solução não é única.

Sds, Ojesed.

- Original Message - 
From: [EMAIL PROTECTED]

To: obm-l@mat.puc-rio.br
Sent: Tuesday, August 15, 2006 5:42 PM
Subject: [obm-l] Invariantes






Ronaldo escreveu:
 Invariância é a propriedade de algo não se alterar quando é submetido a

 uma transformação.  Por exemplo subconjuntos abertos, compactos
  e conexos são invariantes por transformações contínuas
 pois elas levam abertos em abertos

 Cláudio escreveu:

Infelizmente nao eh verdade. A funcao seno eh continua mas leva (0,3pi) em
[-1,1].


   Olá Cláudio. Vou ter que pedir desculpas a lista mais uma vez pois
andei
confundindo os conceitos.

 De fato essa definição é a mais geral de todas: Uma
aplicação de um espaço topológico X em um espaço topológico Y é
contínua se a imagem inverso de um aberto
de Y for aberto em X.   Mas acho que o que eu escrevi sobre compactos
e conexos está correto não está?

   Eu não entendo porque  os livros definem ser aberto  como uma
propriedade topológica já que o exemplo
que você deu mostra que ela não é invariante (a topologia
 de um espaço, por exemplo, é definida como sendo exatamente
a coleção de subconjuntos abertos neste espaço com algumas propriedades
(inclui conjunto vazio e união e intersecção de abertos)) e
propriedades topológicas  são invariantes.  Isso leva a confusões como
as acima.

   A conexidade é importante pelo fato de se conseguir provar que algumas
construções não são possíveis.  Por exemplo a figura abaixo
não pode ser desenhada sem tirar o lápis do papel:

 /\
/  \
   ||
   | \/ |
   | /\ |
   ||

 Acho que uma discussão a respeito de uma demonstração disso pode ajudar
a clarificar o assunto (se alguém conhecer talvez possa publicá-la).

Ronaldo.
=
Instruções para entrar na lista, sair da lista e usar a lista em
http://www.mat.puc-rio.br/~nicolau/olimp/obm-l.html
=


--
No virus found in this incoming message.
Checked by AVG Free Edition.
Version: 7.1.405 / Virus Database: 268.10.10/419 - Release Date: 15/8/2006




=
Instruções para entrar na lista, sair da lista e usar a lista em
http://www.mat.puc-rio.br/~nicolau/olimp/obm-l.html
=


Re: [obm-l] Invariantes

2006-08-15 Por tôpico Ojesed Mirror

Entendi agora, não tem solução mesmo, pois não pode cruzar linha.

 Original Message - 
From: Ojesed Mirror [EMAIL PROTECTED]

To: obm-l@mat.puc-rio.br
Sent: Tuesday, August 15, 2006 11:00 PM
Subject: Re: [obm-l] Invariantes


Sei que este não é o tema central da discussão, mas se eu entendi bem a 
figura, dá pra desenhar-la sem tirar o lápis do papel.
Basta desenhar uma lateral da casa (um traço) e logo após desenhar a parte 
superior do telhado (dois traços), a partir daí qualquer caminho serve.

Esta solução não é única.

Sds, Ojesed.

- Original Message - 
From: [EMAIL PROTECTED]

To: obm-l@mat.puc-rio.br
Sent: Tuesday, August 15, 2006 5:42 PM
Subject: [obm-l] Invariantes






Ronaldo escreveu:
 Invariância é a propriedade de algo não se alterar quando é submetido a

 uma transformação.  Por exemplo subconjuntos abertos, compactos
  e conexos são invariantes por transformações contínuas
 pois elas levam abertos em abertos

 Cláudio escreveu:
Infelizmente nao eh verdade. A funcao seno eh continua mas leva (0,3pi) 
em

[-1,1].


   Olá Cláudio. Vou ter que pedir desculpas a lista mais uma vez pois
andei
confundindo os conceitos.

 De fato essa definição é a mais geral de todas: Uma
aplicação de um espaço topológico X em um espaço topológico Y é
contínua se a imagem inverso de um aberto
de Y for aberto em X.   Mas acho que o que eu escrevi sobre compactos
e conexos está correto não está?

   Eu não entendo porque  os livros definem ser aberto  como uma
propriedade topológica já que o exemplo
que você deu mostra que ela não é invariante (a topologia
 de um espaço, por exemplo, é definida como sendo exatamente
a coleção de subconjuntos abertos neste espaço com algumas propriedades
(inclui conjunto vazio e união e intersecção de abertos)) e
propriedades topológicas  são invariantes.  Isso leva a confusões como
as acima.

   A conexidade é importante pelo fato de se conseguir provar que algumas
construções não são possíveis.  Por exemplo a figura abaixo
não pode ser desenhada sem tirar o lápis do papel:

 /\
/  \
   ||
   | \/ |
   | /\ |
   ||

 Acho que uma discussão a respeito de uma demonstração disso pode ajudar
a clarificar o assunto (se alguém conhecer talvez possa publicá-la).

Ronaldo.
=
Instruções para entrar na lista, sair da lista e usar a lista em
http://www.mat.puc-rio.br/~nicolau/olimp/obm-l.html
=


--
No virus found in this incoming message.
Checked by AVG Free Edition.
Version: 7.1.405 / Virus Database: 268.10.10/419 - Release Date: 
15/8/2006







=
Instruções para entrar na lista, sair da lista e usar a lista em
http://www.mat.puc-rio.br/~nicolau/olimp/obm-l.html
=


Re: Re:[obm-l] Resultado da IMO 2006

2006-07-20 Por tôpico Ojesed Mirror




Achei excelente nosso 
resultado.
É mais fácil melhorar o bronze que 
o ouro, então estamos com vantagem sobre eles.
Com esta equipe, vamos conseguir o ouro mesmo com provas 
"imperfeitas".
Acho que não devemos desmerecer nossos concorrentes pois são todos 
estudantes como os nossos.

Ojesed

  - Original Message - 
  From: 
  claudio.buffara 
  To: obm-l 
  Sent: Wednesday, July 19, 2006 1:04 
  PM
  Subject: Re:[obm-l] Resultado da IMO 
  2006
  
  Antes de mais nada, parabens a nossa equipe! 
  A meu ver, 6 medalhas de bronze mostram muito mais consistencia do que, 
  por exemplo, 1 ouro, 1 prata e 4 maos abanando...
  
  Eu tambem tenho a impressao (por favor me corrijam se eu estiver 
  enganado) de que paises como China e Coreia do Sul preparam seus olimpicos no 
  estilo Kumon, ou seja, fazem cada um deles memorizar centenas (talvez 
  milhares!) de problemas e solucoes para que, na hora da prova, eles dependam 
  mais da memoria do que da criatividade. Isso talvez explique a quantidade de 
  candidatos desses paises que gabaritam as provas da IMO.
  
  Eh claro que as bancas se esforcam pra elaborar problemas queponham 
  a provaa engenhosidade dos candidatos. Mas, como o Gugu me disse uma 
  vez, eh muito facil propor um problema quase impossivel. O dificil eh propor 
  um bom problema que seja resolvivel.Assim, eh possivel que as provas da 
  IMO sejam imperfeitas nesse sentido. Ou seja, se voce tem algum talento 
  matematico (que certamente eh o caso de todos os participantes) e uma 
  preparacao baseada em decoreba intensiva de problemas e metodos de solucao, ha 
  uma boa chance de voce conseguir gabaritar a prova simplesmente por jah ter 
  visto anteriormente alguma questao similar.
  
  []s,
  Claudio.
  
  
  


  De:
  [EMAIL PROTECTED]
  
  


  Para:
  obm-l@mat.puc-rio.br
  
  


  Cópia:
  
  
  


  Data:
  Sun, 16 Jul 2006 
08:52:47 -0700 (PDT)
  
  


  Assunto:
  [obm-l] Resultado 
da IMO 2006
   Oi gente,
   
   Segundo o Mathlinks e mensagens que recebi da equipe
   (via MSN e email) eu tenho a alegria de informá-los
   que toda a equipe do Brasil vai voltar da Eslovênia
   com medalha! Todos ganharam medalha de bronze.
   
   As pontuações são:
   P1 P2 P3 P4 P5 P6 Total
   BRA 1 7 1 0 7 0 0 15
   BRA 2 7 1 0 7 0 0 15
   BRA 3 7 1 1 7 0 0 16
   BRA 4 7 4 0 7 0 0 18
   BRA 5 7 1 1 7 1 0 17
   BRA 6 7 1 0 7 0 0 15
   
   O Brasil ficou em 29o lugar entre os países. Os dez
   primeiros foram (na ordem) China, Rússia, Coréia do
   Sul, Alemanha, EUA, Romênia, Japão, Irã, Moldávia,
   Taiwan.
   
   O Brasil ficou na frente de países como Índia (famosa
   por sua tradição olímpica forte), Suíça, Cazaquistão,
   República Tcheca (que costuma ser forte), boa parte da
   Europa Ocidental (exceções: Reino Unido, Alemanha e
   Itália). Somos o 1o lugar das Américas Central e do
   Sul (o que quer dizer que ganhamos da Argentina,
   hehe). Na América só perdemos para os países da
   América do Norte. Ficamos só 3 posições atrás da
   Bulgária, um país de grande tradição.
   
   Enfim, um resultado que mostra a consistência do
   Brasil na mais importante competição cultural do
   mundo.
   
   Parabéns aos alunos e professores!
   
   []'s
   Shine
   
   PS: Alguém pensou na prova? Vale a pena, é uma das
   melhores IMOs dos últimos anos!
   
   __
   Do You Yahoo!?
   Tired of spam? Yahoo! Mail has the best spam protection around 

   http://mail.yahoo.com 
   
  =
   Instruções para entrar na lista, sair da lista e usar a lista 
  em
   http://www.mat.puc-rio.br/~nicolau/olimp/obm-l.html
   
  =
   
  
  

  No virus found in this incoming message.Checked by AVG Free 
  Edition.Version: 7.1.394 / Virus Database: 268.10.2/393 - Release Date: 
  19/7/2006


Re: [obm-l] Calculo Numerico.

2006-06-27 Por tôpico Ojesed Mirror

o número é o que satisfaz a equação cosx=x
só conheço método numérico para encontrá-lo.

Ojesed.


- Original Message - 
From: [EMAIL PROTECTED]

To: obm-l@mat.puc-rio.br
Sent: Tuesday, June 27, 2006 8:26 AM
Subject: [obm-l] Calculo Numerico.


Favor quem pode me ajudar com esta questão.

Em uma calculadora cientifica, ajuste a medida de ângulos como sendo 
radianos

e digite aleatoriamente um número qualquer. Pressione a tecla da função
cosseno varias vezes (no mínimo 20 vezes). Com isso, não importa qual tenha
sido o numero digitado aleatoriamente no inicio, a seqüência de números
que vão aparecendo no visor á medida que a tecla da função cosseno é 
pressionada

converge sempre para o mesmo numero.

Identifique que número é esse e por qual motivo ele sempre aparece.

Obrigado.




=
Instruções para entrar na lista, sair da lista e usar a lista em
http://www.mat.puc-rio.br/~nicolau/olimp/obm-l.html
=


--
No virus found in this incoming message.
Checked by AVG Free Edition.
Version: 7.1.394 / Virus Database: 268.9.2/373 - Release Date: 22/6/2006


=
Instruções para entrar na lista, sair da lista e usar a lista em
http://www.mat.puc-rio.br/~nicolau/olimp/obm-l.html
=


[obm-l] Re: [obm-l] Função Gamma.

2006-06-26 Por tôpico Ojesed Mirror



Perece que no matlab, gamma com 
dois argumentos é a função gamado segundo argumento, com os limites de 
integração de zero até o primeiro argumento, dividido por gama do segundo 
argumento.

Não sei se é isto...


  - Original Message - 
  From: 
  Ronaldo Luiz 
  Alonso 
  To: obm-l@mat.puc-rio.br 
  Sent: Monday, June 26, 2006 10:36 
AM
  Subject: [obm-l] Função Gamma.
  Olá Ojesed: Pelo Matlab a resposta seria: 
  x*(pi*2^(1/2)-gamma(1/4,-x^4)*gamma(3/4)) 
  - 4*gamma(3/4)*(-x^4)^(1/4) 
  Deve ter algum problema com: gamma(1/4,-x^4) pois que eu me lembre a função 
  gamma é uma função de 1 variável apenas... P.S.I, 
  200.153.238.168, sent you this email using www.Fake-Mailer.com This email is 
  fake. 
  = 
  Instruções para entrar na lista, sair da lista e usar a lista em 
  http://www.mat.puc-rio.br/~nicolau/olimp/obm-l.html 
  =
  
  

  No virus found in this incoming message.Checked by AVG Free 
  Edition.Version: 7.1.394 / Virus Database: 268.9.2/373 - Release Date: 
  22/6/2006


math_g2a.gif
Description: GIF image


Re: Re:[obm-l]- Integral

2006-06-25 Por tôpico Ojesed Mirror



Me tirem uma dúvida por 
favor.
Porque a pergunta original é sobre 
uma integral indefinida e as respostas estão se referindo a somatório 
?
Pelo Matlab a resposta 
seria:

x*(pi*2^(1/2)-gamma(1/4,-x^4)*gamma(3/4))

4*gamma(3/4)*(-x^4)^(1/4)

Ojesed.

  - Original Message - 
  From: 
  Marcelo Salhab 
  Brogliato 
  To: obm-l@mat.puc-rio.br 
  Sent: Friday, June 23, 2006 4:03 AM
  Subject: Re: Re:[obm-l]- Integral
  
  Olá,
  apenas alguns detalhes..
  
  e^(x^4) = somatorio de n=0 até infinito de 
  x^(4n)/n!
  esta serie converge para todo x real, e é uma 
  série de potências, deste modo, ela é uniformemente convergente para todo x 
  real, e podemos dizer
  que o integral da serie é a serie da integral... 
  deste modo, temos:
  
  integral e^(x^4) = somatorio de n=0 até infinito 
  de x^(4n+1)/[n! (4n+1)]
  
  esta integral convergepara todo x 
  real.
  
  abraços,
  Salhab
  
  
- Original Message - 
From: 
Giuliano (stuart) 
To: obm-l 
Sent: Thursday, June 22, 2006 3:13 
PM
Subject: Re:[obm-l]- Integral


Bom Dia! 
sabemos que e^x=somatório de n=0 até infinito de (x^n)/n!
mas comoô que vc queré
e^(x^4) =somatório de n=0 até infinito de (x^4n)/n! logo a 
integral será
somatório de n=1 até infinito de (x^(4n-1))/(n!*4n) 






 O pessoal, to precisando de uma luz aqui numa questão 
 Qual é a integral de e^(x^4) dx ? 
 isso se essa primitiva realmente existir 
 
 Obrigado 
 

Abraços, 
Giuliano Pezzolo Giacaglia 
(Stuart)
  
  

  No virus found in this incoming message.Checked by AVG Free 
  Edition.Version: 7.1.394 / Virus Database: 268.9.2/373 - Release Date: 
  22/6/2006


Re: [obm-l] sequencia basica

2006-06-14 Por tôpico Ojesed Mirror



n/2^(n-1)

  - Original Message - 
  From: 
  Johann Peter Gustav Lejeune 
  Dirichlet 
  To: obm-l@mat.puc-rio.br 
  Sent: Tuesday, June 13, 2006 12:19 
  PM
  Subject: Re: [obm-l] sequencia 
  basica
  Sem querer ser chato, diga-me qual a lei de formacao 
  disto...
  Em 06/06/06, Eduardo 
  Soares [EMAIL PROTECTED]  
  escreveu: 
  


1 + 2/2 + 3/4 + 4/8 + 5/16 ... = 


Ligações gratuitas de PC-para-PC para qualquer lugar do Brasil e do mundo 
com o MSN Messenger. Saiba mais 
em: 
= 
Instruções para entrar na lista, sair da lista e usar a lista em http://www.mat.puc-rio.br/~nicolau/olimp/obm-l.html 
= 
-- Ideas are 
  bulletproof.V 
  
  

  No virus found in this incoming message.Checked by AVG Free 
  Edition.Version: 7.1.394 / Virus Database: 268.8.4/363 - Release Date: 
  13/6/2006


[obm-l] Re: [obm-l] Triângulos Pitagóricos (was:12^2 + 3 3^2 = 1233^2)

2006-06-09 Por tôpico Ojesed Mirror

Preciosidade vamos acalmar com calma, muito bom, vou usar muito.

- Original Message - 
From: [EMAIL PROTECTED]

To: obm-l@mat.puc-rio.br
Sent: Friday, June 09, 2006 3:33 PM
Subject: [obm-l] Triângulos Pitagóricos (was:12^2 + 33^2 = 1233^2)




Oi pessoal, vamos acalmar com calma:
  Espero que essa mensagem possa ajudar neste problema (embora
possa como todas as minhas outras possa
ser apenas um pitaco sem nenhuma utilidade).

Sabemos que:
(n^2 - 1)^2 + (2n)^2 = (n^2 +1)^2

  para n natural, n1 ela dá todos os triângulos pitagóricos.
  Ex: n=2 : 3^2 + 4^2 = 5^2 .
   A intenção é usar essa identidade para tentar obter quadrados
perfeitos naturais da forma Delta^2 = b^2 - 4ac.
  Neste caso usamos:
 (n^2 - 1)^2  = (n^2 +1)^2  - (2n)^2
   (n^2 - 1)^2 = (n^2 +1)^2 - 4 n^2

   Supondo a = 1 (sempre dá para fazer a=1 em uma eq. do 2 grau).
 Temos então que ter:
   b = n^2 +1
   c= n^2   == b = c+1

Bom... agora será que dá para aplicar isso à equação em jogo?


100a+b = a^2 + b^2
basta resolver essa eq de 2º grau com relação a a
e temos
a = 50 +- sqrt(2500+b-b^2)


  Para não causar confusão vamos trocar a por x e b por y:

  100x + y = x^2 + y^2

  x^2 -100x +y -y^2 = 0

  Construindo o Delta:
   Delta^2 = 100^2 - 4*(y-y^2)

com b = 100 e c = y-y^2
como b= c+1
100 = y-y^2 +1

  Quais y naturais com 2 algarismos verificam isso?




=
Instruções para entrar na lista, sair da lista e usar a lista em
http://www.mat.puc-rio.br/~nicolau/olimp/obm-l.html
=


--
No virus found in this incoming message.
Checked by AVG Free Edition.
Version: 7.1.394 / Virus Database: 268.8.3/360 - Release Date: 9/6/2006




=
Instruções para entrar na lista, sair da lista e usar a lista em
http://www.mat.puc-rio.br/~nicolau/olimp/obm-l.html
=


Re: [obm-l] Integral de sen^3(x)*cos(x)dx

2006-06-01 Por tôpico Ojesed Mirror



Se y=a*f(x)^n sabe-se que y' = 
n*a*f(x)^(n-1)*f '(x).

No seu caso a=1/4, n=4 
f(x)=sen(x)
É só substituir que sai 
direto.


  - Original Message - 
  From: 
  Camilo 
  Damiao 
  To: obm-l@mat.puc-rio.br 
  Sent: Thursday, June 01, 2006 10:36 
  PM
  Subject: Re: [obm-l] Integral de 
  sen^3(x)*cos(x)dx
  Intaum keh dize q a integral sai direto?Sem nenhuma 
  substituiçaum trigonometrica???
  
  

  No virus found in this incoming message.Checked by AVG Free 
  Edition.Version: 7.1.394 / Virus Database: 268.8.0/353 - Release Date: 
  31/5/2006


[obm-l] Re: [obm-l] Rotação em torno de um eixo arbitrár io.

2006-05-11 Por tôpico Ojesed Mirror



boa sorte...

  - Original Message - 
  From: 
  Ronaldo Luiz 
  Alonso 
  To: obm-l@mat.puc-rio.br 
  Sent: Thursday, May 11, 2006 8:58 
PM
  Subject: [obm-l] Rotação em torno de um 
  eixo arbitrário.
  
  Achei... mas é confuso ...:
  
  http://www.mines.edu/~gmurray/ArbitraryAxisRotation/ArbitraryAxisRotation.html
  
  

  No virus found in this incoming message.Checked by AVG Free 
  Edition.Version: 7.1.392 / Virus Database: 268.5.6/337 - Release Date: 
  11/5/2006


[obm-l] Re: [obm-l] problema do almoço

2006-05-09 Por tôpico Ojesed Mirror


Sendo 60 min o intervalo de chegada e 10 min o tempo de espera para ambos 
temos:


A probabilidade deles não se encontrarem é (60-10)^2/60^2 = 69,44%
A probabilidade de se encontrárem é 1 - 69,44% = 30,56%
Para detalhes, veja a página 35 do livro do Papoulis que explora bem este 
tema.


Ojesed.

- Original Message - 
From: Fabio Silva [EMAIL PROTECTED]

To: obm-l@mat.puc-rio.br
Sent: Tuesday, May 09, 2006 11:06 AM
Subject: [obm-l] problema do almoço


Srs, peço ajuda na resolução deste problema:



Dois amigos combinaram um encontro para almoçar entre
12:00 e 13:00h. Alguns dias depois, ambos esquecem o
momeno exato, mas nenhum deles desiste de ir ao
encontro, e ambos resolvem ir ao encontro escolhendo a
hora de chegar aleatoreamente (e independentemente)
entre 12:00 e 13:00h. Se cada um deles desiste
esperar, no máximo, 10 min, qual a probabilidade dos
dois amigos almoçarem juntos neste dia?(eles não tem
celular!)


__
Do You Yahoo!?
Tired of spam?  Yahoo! Mail has the best spam protection around
http://mail.yahoo.com
=
Instruções para entrar na lista, sair da lista e usar a lista em
http://www.mat.puc-rio.br/~nicolau/olimp/obm-l.html
=


--
No virus found in this incoming message.
Checked by AVG Free Edition.
Version: 7.1.392 / Virus Database: 268.5.5/334 - Release Date: 8/5/2006


=
Instruções para entrar na lista, sair da lista e usar a lista em
http://www.mat.puc-rio.br/~nicolau/olimp/obm-l.html
=


[obm-l] Re: [obm-l] ESQUEÇAM A ÁLGEBRA!

2006-05-03 Por tôpico Ojesed Mirror
Com esta forma coloquial de lidar com matemática só dá pra resolver 
problemas de ovo galinha e filhos.
Certamente este não é o objetivo dela e ainda bem que a muito tempo os 
homens entenderam que a linguagem simbólica é mais eficiente.
Sua abordagem vale como diversão e acho que com este objetivo é muito 
válida, afinal sem diversão a vida seria muito chata.

Lembra dos algarismos romanos ?
Lembra porque eles foram abandonados ?

Ojesed.

- Original Message - 
From: Jorge Luis Rodrigues e Silva Luis [EMAIL PROTECTED]

To: obm-l@mat.puc-rio.br
Sent: Wednesday, May 03, 2006 8:53 AM
Subject: [obm-l] ESQUEÇAM A ÁLGEBRA!


Calma! Apenas frações e contas em lugar de equações e incógnitas, pois nem 
só de xis vive a matemática. Muitas vezes, formas não matemáticas de 
resolver um problema podem ser mais elegantes e econômicas.


Uma senhora resolve presentear os três filhos com ovos frescos que traz em 
uma cesta. Ao mais velho, ela dá a metade do que possui e mais meio ovo. O 
do meio recebe metade do que restou na cesta e mais meio ovo. O filho mais 
novo ganha a metade do novo resto e mais meio ovo e a mãe fica sem nada. 
Quantos ovos havia na cesta e quantos a mãe deu a cada filho?


Raciocinando do terceiro para o primeiro filho: se a mãe deu ao último 
filho metade do que havia na cesta e mais meio ovo, ficando sem nada, é 
porque meio ovo é a metade do conteúdo da cesta. Logo, havia um ovo apenas 
quando ela chegou ao terceiro filho. Bem, já sabemos que o terceiro 
recebeu apenas um ovo, que era tudo o que a mãe possuia ao deixar o 
segundo filho. Ora, ao segundo filho, ela deu a metade do que havia 
sobrado do primeiro e mais meio ovo, ficando, como já sabemos, com um 
único ovo. Se não tivesse dado o meio ovo, teria lhe restado um ovo e 
meio, ou seja, metade da quantidade com que chegou ao segundo filho: três 
ovos. Observe que, se chegou ao segundo com três ovos, deu metade da 
quantidade (um e meio) e mais meio: dois ovos portanto. Assim, já sabemos 
que ao terceiro ela deu um ovo e ao segundo, dois, tendo vindo do primeiro 
filho com três ovos. Isso é a metade do total, menos meio ovo; portanto, a 
metade do conteúdo total da cesta é equivalente a três ovos e meio. 
Consequentemente, a mãe possuia sete ovos. Ao primeiro filho deu a metade 
(três e meio) e mais meio = quatro ovos.


Da mesma forma como registramos o uso abusivo do computador e das 
calculadoras no ensino fundamental, tem muita gente que quando as esquece 
em casa tem a sensação de ter saído sem cérebro. O mesmo se aplica a 
certos currículos que, de tão algebrizados sonegam aos alunos a Aritmética 
e a Geometria.


Abraços!

_
COPA 2006: O horário dos jogos do Brasil na Copa Clique aqui! 
http://copa.br.msn.com/tabelas/tabela/


=
Instruções para entrar na lista, sair da lista e usar a lista em
http://www.mat.puc-rio.br/~nicolau/olimp/obm-l.html
=


--
No virus found in this incoming message.
Checked by AVG Free Edition.
Version: 7.1.385 / Virus Database: 268.5.1/328 - Release Date: 1/5/2006




=
Instruções para entrar na lista, sair da lista e usar a lista em
http://www.mat.puc-rio.br/~nicolau/olimp/obm-l.html
=


Re: [obm-l] LIMITES

2006-05-01 Por tôpico Ojesed Mirror



a) Fazendo x=1/y quando x-0+ 
y-+inf.
x^x = (1/y)^(1/y) = 
exp(-ln(y)/y)
Observe que y cresce mais rápido 
que ln(y), logo o expoente tende a zero e o limite de x^x tende a 1

Ojesed.

  - Original Message - 
  From: 
  Klaus 
  Ferraz 
  To: obm-l@mat.puc-rio.br 
  Sent: Friday, April 28, 2006 2:42 
PM
  Subject: [obm-l] LIMITES
  
  a) lim(x-0+) x^x
  b)lim(x-a) (x^n-a^n)/((lnx)^n-(lna)^n)
  
  
  
  Abra 
  sua conta no Yahoo! Mail - 1GB de espaço, alertas de e-mail no celular e 
  anti-spam realmente eficaz. 
  
  

  No virus found in this incoming message.Checked by AVG Free 
  Edition.Version: 7.1.385 / Virus Database: 268.5.1/326 - Release Date: 
  27/4/2006


Re: [obm-l] Duvidas

2006-04-21 Por tôpico Ojesed Mirror



Como a mistura final deve estar a 
5%, considerando "A"a quantidade de água acrescida e "V" a quantidade de 
vinhoacrescida temos:

(300+A+V)/V = 1/5% = 20 


300/V + (A+V)/V = 
20

Pelo enunciado (A+V)/V = 1/12,5% = 
8(I)

300/V + 8 = 20

V = 25

De (I) tiramos A = 175

V + A = 200, resposta letra 
E.


  - Original Message - 
  From: 
  matduvidas48 
  To: obm-l 
  Sent: Friday, April 21, 2006 7:10 
AM
  Subject: [obm-l] Duvidas
  
  
  
  
  
  Alguém poderia resolver esta questão.
  Quantos ml de vinho tinto, com percentual alcoólico de
  12,5%, devem ser diluídos com 300ml de água, de
  modo a se obter uma mistura com percentual alcoólico
  de 5%?
  A) 120ml
  B) 140ml
  C) 160ml
  D) 180ml
  E) 200ml
  
  Aline Marques
  
  

  No virus found in this incoming message.Checked by AVG Free 
  Edition.Version: 7.1.385 / Virus Database: 268.4.4/319 - Release Date: 
  19/4/2006


[obm-l] Re: [obm-l] Re: [obm-l] Re: [obm-l] Inequa ção entre função quadrática e exponencial

2006-04-20 Por tôpico Ojesed Mirror

Talvez se colocar mais um termo do polinômio dê pra achar a terceira raiz.
Parece que esta aproximação só é boa perto da origem, como a outra raiz é 
próxima de 14 não sei como ficará a precisão.

Vou tentar fazer.

- Original Message - 
From: Marcelo Salhab Brogliato [EMAIL PROTECTED]

To: obm-l@mat.puc-rio.br
Sent: Thursday, April 20, 2006 3:20 AM
Subject: [obm-l] Re: [obm-l] Re: [obm-l] Inequação entre função quadrática e 
exponencial



Olá,

seja f(x) = 100n^2 - 2^n, entao:

por inspeção:
n=0: f(x) = -1
n=1: f(x) = 98
n=-1:f(x) = 99,5

deste modo, estes pontos tem de estar entre -1 e 1.

então, talvez uma boa aproximacao seria por polinomio de taylor, onde
teremos:

f(x) = 100n^2 - e^[n*ln(2)]

e^x = 1 + x + x^2/2 + ...

logo: f(x) = 100n^2 - (1 + n*ln(2) + n^2 * [ln(2)]^(2) / 2)

cujas raizes sao: 0,1037 e -0,0967... (pode calcular por baskara)

espero ter ajudado,
abraços,
Salhab

- Original Message - 
From: Ojesed Mirror [EMAIL PROTECTED]

To: obm-l@mat.puc-rio.br
Sent: Thursday, April 20, 2006 12:57 AM
Subject: [obm-l] Re: [obm-l] Inequação entre função quadrática e exponencial


-0.00996552170823 e 22.23756639530996 considerando (100*n)^2  2^n
-0.09670403432670 e 14.32472783699820 considerando 100*(n^2)  2^n

Acho que não tem método analítico de resolução, se tiver quero conhecer.

Ojesed.

- Original Message - 
From: Henrique Rennó [EMAIL PROTECTED]

To: obm-l@mat.puc-rio.br
Sent: Wednesday, April 19, 2006 11:14 PM
Subject: [obm-l] Inequação entre função quadrática e exponencial


Olá pessoal da lista!!!

Gostaria de saber uma possível solução para o problema:

100n^2  2^n

Se verificarmos pelos gráficos das duas funções 100n^2 e 2^n
sobrepostos, existem dois pontos que limitam uma região onde a função
100n^2 é menor que 2^n. Quais são os dois valores de n que limitam
essa região?

Agradeço a atenção,

Abraços!!!

--
Henrique
Não há ninguém que seja tão grande que não possa aprender e nem tão
pequeno que não possa ensinar.
There's no one that is so great that could not learn nor so small
that could not teach.
O indivíduo confiante tenta mais, erra mais, aprende mais. - Piaget
The confident individual try more, err more, learn more. - Piaget

=
Instruções para entrar na lista, sair da lista e usar a lista em
http://www.mat.puc-rio.br/~nicolau/olimp/obm-l.html
=


--
No virus found in this incoming message.
Checked by AVG Free Edition.
Version: 7.1.385 / Virus Database: 268.4.3/317 - Release Date: 18/4/2006


=
Instruções para entrar na lista, sair da lista e usar a lista em
http://www.mat.puc-rio.br/~nicolau/olimp/obm-l.html
=

=
Instruções para entrar na lista, sair da lista e usar a lista em
http://www.mat.puc-rio.br/~nicolau/olimp/obm-l.html
=


--
No virus found in this incoming message.
Checked by AVG Free Edition.
Version: 7.1.385 / Virus Database: 268.4.3/317 - Release Date: 18/4/2006


=
Instruções para entrar na lista, sair da lista e usar a lista em
http://www.mat.puc-rio.br/~nicolau/olimp/obm-l.html
=


[obm-l] Re: [obm-l] Inequação entre função quadr ática e exponencial

2006-04-19 Por tôpico Ojesed Mirror

-0.00996552170823 e 22.23756639530996 considerando (100*n)^2  2^n
-0.09670403432670 e 14.32472783699820 considerando 100*(n^2)  2^n

Acho que não tem método analítico de resolução, se tiver quero conhecer.

Ojesed.

- Original Message - 
From: Henrique Rennó [EMAIL PROTECTED]

To: obm-l@mat.puc-rio.br
Sent: Wednesday, April 19, 2006 11:14 PM
Subject: [obm-l] Inequação entre função quadrática e exponencial


Olá pessoal da lista!!!

Gostaria de saber uma possível solução para o problema:

100n^2  2^n

Se verificarmos pelos gráficos das duas funções 100n^2 e 2^n
sobrepostos, existem dois pontos que limitam uma região onde a função
100n^2 é menor que 2^n. Quais são os dois valores de n que limitam
essa região?

Agradeço a atenção,

Abraços!!!

--
Henrique
Não há ninguém que seja tão grande que não possa aprender e nem tão
pequeno que não possa ensinar.
There's no one that is so great that could not learn nor so small
that could not teach.
O indivíduo confiante tenta mais, erra mais, aprende mais. - Piaget
The confident individual try more, err more, learn more. - Piaget

=
Instruções para entrar na lista, sair da lista e usar a lista em
http://www.mat.puc-rio.br/~nicolau/olimp/obm-l.html
=


--
No virus found in this incoming message.
Checked by AVG Free Edition.
Version: 7.1.385 / Virus Database: 268.4.3/317 - Release Date: 18/4/2006


=
Instruções para entrar na lista, sair da lista e usar a lista em
http://www.mat.puc-rio.br/~nicolau/olimp/obm-l.html
=


Re: [obm-l] Armazenamento de Matriz em Vetor

2006-04-11 Por tôpico Ojesed Mirror

Para calcular a linha use i=(sqrt(8*pos + 9)-3)/2 e arredonde para cima.
Para calcular a coluna use a formula que calcula pos.

Ojesed.

- Original Message - 
From: Henrique Rennó [EMAIL PROTECTED]

To: obm-l@mat.puc-rio.br
Sent: Tuesday, April 11, 2006 11:11 AM
Subject: [obm-l] Armazenamento de Matriz em Vetor


Olá pessoal da lista!!!

Estou com um problema para representar uma matriz na forma de um
vetor. Suponha que tenhamos uma matriz diagonal inferior:

1 0 0 0 0
6 3 0 0 0
4 2 6 0 0
8 3 7 1 0
9 5 2 7 4

Não há necessidade de representar essa matriz armazenando todos os
elementos, pois sabemos que acima da diagonal principal todos os
elementos são zero. Dessa forma, podemos armazenar apenas os elementos
da diagonal principal e da parte inferior seguindo a seguinte regra:

pos = [(1+i)*i]/2 + j

onde pos é a posição no vetor e a posição na matriz inicia em i = 0, j
= 0 (i = linha, j = coluna). Uma matriz de dimensão N tem seus
elementos na forma:

0,0 0,1 0,2 ... 0,N-1
1,0 1,1 1,2 ... 1,N-1
 .....
 .....
 .....
N-1,0 N-1,1 ... N-1,N-1

Por exemplo, se i = 3 e j = 2, pos seria 8 e o elemento 7 estaria
nesta posição. (O vetor se inicia na posição zero - 0).

O problema é saber uma forma de, conhecendo pos (a posição do elemento
no vetor), saber quais os valores de i e j (linha e coluna) do
elemento na matriz.

Estou com essa dúvida porque estou desenvolvendo um programa em C para
mexer com matrizes que estão nessa forma (os elementos acima da
diagonal superior são todos zeros) e para dimensões maiores, por
exemplo 1, teríamos um número imenso de zeros ocupando memória sem
necessidade.

Obs: caso a matriz possua todos os elementos nulos abaixo da diagonal
principal, acredito que trocando i por j e j por i na igualdade acima
para o cálculo de pos funciona normalmente, mas o que preciso mesmo
não é este caso.

Agradeço a atenção de todos,

Abraços!!!

--
Henrique

=
Instruções para entrar na lista, sair da lista e usar a lista em
http://www.mat.puc-rio.br/~nicolau/olimp/obm-l.html
=


--
No virus found in this incoming message.
Checked by AVG Free Edition.
Version: 7.1.385 / Virus Database: 268.4.1/307 - Release Date: 10/4/2006


=
Instruções para entrar na lista, sair da lista e usar a lista em
http://www.mat.puc-rio.br/~nicolau/olimp/obm-l.html
=


Re: [obm-l] Re:Equacao exponencial simples

2006-04-09 Por tôpico Ojesed Mirror



Muito 
bomEduardo.
Me tire uma dúvida, se esta raiz foi achada por inspeção, não seria 
necessário provar que ela é única para completar a solução ?

- Original Message - 

  From: 
  Eduardo Wilner 
  To: obm-l@mat.puc-rio.br 
  Sent: Sunday, April 09, 2006 11:22 
  AM
  Subject: [obm-l] Re:Equacao exponencial 
  simples
  
  3^(x/2) + 1 = 4^(x/2) ,  
  3 + 1 = 4 = x/2 =1 ou  x=2em nome da 
  simpicidade ?.
  - Original Message 
  --
  RONALD MARTINSSat, 08 Apr 
  2006 06:07:49 -0700
  Como resolver, de forma simples, a equacao 3^(x/2) + 1 = 2^x ?
   
  x/2 
  x3 + 1 = 2
  
  Abraço a todos.Ronald. 
  
  
  Yahoo! SearchImposto de Renda 2006: o prazo está acabando. Faça 
  já a sua declaração no site da Receita Federal.
  
  

  No virus found in this incoming message.Checked by AVG Free 
  Edition.Version: 7.1.385 / Virus Database: 268.4.0/305 - Release Date: 
  8/4/2006


[obm-l] Re: [obm-l] Re: [obm-l] Re: [obm-l] dúvi da fatorial

2006-04-04 Por tôpico Ojesed Mirror
Não é convenção pelo simples fato de que não se pode atribuir outro valor a 
estes fatoriais, sem causar inconsistências.
Uma convenção pode ser mudada sem causar conflitos e este certamente não é o 
caso.


Como estes fatoriais não podem assumir nenhum outro valor, então existe uma 
obrigatoriedade nesta atribuição e convenção não é obrigatória mas apenas 
conveniente.


Ojesed

- Original Message - 
From: Ronaldo Luiz Alonso [EMAIL PROTECTED]

To: obm-l@mat.puc-rio.br
Sent: Tuesday, April 04, 2006 9:51 AM
Subject: [obm-l] Re: [obm-l] Re: [obm-l] dúvida fatorial


Qualquer valor diferente de um atribuído por convenção estaria negando 
a definição de fatorial.


SE considerarmos  a interpretação de fatorial
como número de bijeções de um conjunto com n
elementos em um conjunto com n elementos
e SE considerarmos a definição de números
binomiais em termos de fatorial como usualmente
nos são apresentadas, aí podemos dizer que o que
vc escreveu está correto. Não há como trocar
a definição sem causar conflitos.

Mas todas essas definições são, de fato, convenções.
Então a definição 0! = 1 também (do mesmo jeito)
é uma convenção.
Não consigo ver como não seria com o que
nos foi apresentado até agora  ...




Ojesed.

- Original Message - 
From: Nicolau C. Saldanha [EMAIL PROTECTED]

To: obm-l@mat.puc-rio.br
Sent: Monday, April 03, 2006 4:19 PM
Subject: Re: [obm-l] dúvida fatorial


On Mon, Apr 03, 2006 at 09:49:58AM -0300, reginaldo.monteiro wrote:

Alguém saberia me informar por que 0! = 1?


Alguém já respondeu corretamente que isto é uma convenção,
mas acho que há mais para ser dito.

A interpretação combinatória para n! é que este é o número
de permutações de um conjunto A com n elementos. Recapitulando,
uma permutação de A é uma função bijetora f:A-A, ou,
equivalentemente, um subconjunto F de AxA (o gráfico de f)
tal que, para todo a em A:
* existe um único b em A tal que (a,b) pertence a F;
* existe um único c em A tal que (c,a) pertence a F.

Com esta definição, se A = 0 (vazio) então F = 0 é o gráfico
de uma bijeção f:A-A, a função vazia. As condições para verificar
que f é bijetora são satisfeitas por vacuidade. É bem claro
que esta é a única permutação de A, donde 0!=1.

[]s, N.
=
Instruções para entrar na lista, sair da lista e usar a lista em
http://www.mat.puc-rio.br/~nicolau/olimp/obm-l.html
=


--
No virus found in this incoming message.
Checked by AVG Free Edition.
Version: 7.1.385 / Virus Database: 268.3.4/299 - Release Date: 31/3/2006


=
Instruções para entrar na lista, sair da lista e usar a lista em
http://www.mat.puc-rio.br/~nicolau/olimp/obm-l.html
=



=
Instruções para entrar na lista, sair da lista e usar a lista em
http://www.mat.puc-rio.br/~nicolau/olimp/obm-l.html
=


--
No virus found in this incoming message.
Checked by AVG Free Edition.
Version: 7.1.385 / Virus Database: 268.3.4/299 - Release Date: 31/3/2006

=
Instruções para entrar na lista, sair da lista e usar a lista em
http://www.mat.puc-rio.br/~nicolau/olimp/obm-l.html
=


[obm-l] Re: [obm-l] Número de Algarismos do Produto

2006-04-04 Por tôpico Ojesed Mirror



16^13,25 = 2^53 = 
8*(2^50)
25^25 = 5^50

8*(2^50)*(5^50) = 8*10^50 - 51 
algarismos.

  - Original Message - 
  From: 
  Rhilbert Rivera 
  To: obm-l@mat.puc-rio.br 
  Sent: Tuesday, April 04, 2006 7:01 
  PM
  Subject: [obm-l] Número de Algarismos do 
  Produto
  
  
  Amigos, alguém pode me dar uma grande ajuda neste problema?
  " Determinar o número de algarismos do produto 16^13,25 x 25^25"
  Pessoal, existe algum resultado que garanta o número de algarismos do 
  produto entre dois números?
  Obrigado!
  
  COPA 2006: O horário dos jogos do Brasil na Copa Clique aqui: 
  = 
  Instruções para entrar na lista, sair da lista e usar a lista em 
  http://www.mat.puc-rio.br/~nicolau/olimp/obm-l.html 
  =
  
  

  No virus found in this incoming message.Checked by AVG Free 
  Edition.Version: 7.1.385 / Virus Database: 268.3.4/299 - Release Date: 
  31/3/2006


[obm-l] Re: [obm-l] dúvida fatorial

2006-04-03 Por tôpico Ojesed Mirror

Por definição n! = n*(n-1)! para n natural maior que 1.
Se fizermos n=2 deduzimos que 1!=1
Se fizermos n=1 deduzimos que 0!=1

Então, 0! e 1! são iguais a um por extensão/conseqüência da definição de 
fatorial e não por convenção.
Qualquer valor diferente de um atribuído por convenção estaria negando a 
definição de fatorial.


Ojesed.

- Original Message - 
From: Nicolau C. Saldanha [EMAIL PROTECTED]

To: obm-l@mat.puc-rio.br
Sent: Monday, April 03, 2006 4:19 PM
Subject: Re: [obm-l] dúvida fatorial


On Mon, Apr 03, 2006 at 09:49:58AM -0300, reginaldo.monteiro wrote:

Alguém saberia me informar por que 0! = 1?


Alguém já respondeu corretamente que isto é uma convenção,
mas acho que há mais para ser dito.

A interpretação combinatória para n! é que este é o número
de permutações de um conjunto A com n elementos. Recapitulando,
uma permutação de A é uma função bijetora f:A-A, ou,
equivalentemente, um subconjunto F de AxA (o gráfico de f)
tal que, para todo a em A:
* existe um único b em A tal que (a,b) pertence a F;
* existe um único c em A tal que (c,a) pertence a F.

Com esta definição, se A = 0 (vazio) então F = 0 é o gráfico
de uma bijeção f:A-A, a função vazia. As condições para verificar
que f é bijetora são satisfeitas por vacuidade. É bem claro
que esta é a única permutação de A, donde 0!=1.

[]s, N.
=
Instruções para entrar na lista, sair da lista e usar a lista em
http://www.mat.puc-rio.br/~nicolau/olimp/obm-l.html
=


--
No virus found in this incoming message.
Checked by AVG Free Edition.
Version: 7.1.385 / Virus Database: 268.3.4/299 - Release Date: 31/3/2006


=
Instruções para entrar na lista, sair da lista e usar a lista em
http://www.mat.puc-rio.br/~nicolau/olimp/obm-l.html
=


Re: Re:[Desejados] Re: [obm-l] Ajuda

2006-04-03 Por tôpico Ojesed Mirror



faça 2+sqrt(3)=a, 
logo2-sqrt(3)=1/a
faça a^(x/2)=y (I)

você ficará com y + 1/y = 4 - 
y1=a, y2=1/a

substituindo em (I) 
temos

a^(x/2) = a - x=2
a^(x/2) = 1/a - 
x=-2

Ojesed

  - Original Message - 
  From: 
  andre.pereira 
  To: obm-l 
  Cc: obm-l 
  Sent: Monday, April 03, 2006 12:12 
  PM
  Subject: Re:[Desejados] Re: [obm-l] 
  Ajuda
  
  
  
  


  De:
  [EMAIL PROTECTED]
  
  


  Para:
  obm-l@mat.puc-rio.br
  
  


  Cópia:
  
  
  


  Data:
  Mon, 3 Apr 2006 
01:06:55 -0300
  
  


  Assunto:
  [Desejados] Re: 
[obm-l] Ajuda
  Marcelo, agradeço imensamente sua ajuda, mas eu cometi um erro quando 
  digitei a equação (não estou acostumado com essa convenção), que corretamente 
  deveria ser:
  
  SQRT((2+SQRT(3)) ^ x) + SQRT((2-SQRT(3)) ^ x) = 4 
  
  
  Estou seguindo o seguinte caminho para tentar a resolução:
  
  (2+sqrt(3))^x + 2(2+sqrt(3))^x*(2-sqrt(3))^x + (2-sqrt(3))^x=16
  
  (2+sqrt(3))^x + (2-sqrt(3))^x + 2^x=16 ... e parei por aqui!
  
  Não sei se é esse o raciocínio.
  
  Obrigado
  
  
  
   Olá,
   
   sqrt(3)^x = y
   
   logo: sqrt(2+y) + sqrt(2-y) = 4
   2 + y + 2 - y + 2sqrt(4-y^2) = 
  16
   sqrt(4-y^2) = 6
   4-y^2 = 36
   y^2 = -32
   absurdo! pois y^2  0 
  sempre...
   logo, nao tem solucao
   
   abraços,
   Salhab
  
- Original Message - 
From: André 
To: obm-l@mat.puc-rio.br 
Sent: Sunday, April 02, 2006 10:25 PM
Subject: [obm-l] Ajuda
 

Alguem poderia me ajudar a 
resolver a equacao abaixo, sou novo por aqui e nao sei se posso pedir ajuda 
aqui. 
Obrigado e 
desculpe.

SQRT(2+SQRT(3) 
^ x) + SQRT(2 - SQRT(3) ^ x) = 4 
  
  
  E-mail classificado pelo Identificador 
  de Spam Inteligente.Para alterar a categoria classificada, visite o Terra 
  Mail 
  
  
  
  Esta mensagem foi verificada pelo E-mail Protegido Terra.Scan engine: 
  McAfee VirusScan / Atualizado em 31/03/2006 / Versão: 4.4.00/4731Proteja o 
  seu e-mail Terra: http://mail.terra.com.br/ 
  
  
  

  No virus found in this incoming message.Checked by AVG Free 
  Edition.Version: 7.1.385 / Virus Database: 268.3.4/299 - Release Date: 
  31/3/2006